Vision: Www.visionias.in

  • Uploaded by: Monti Dabas
  • 0
  • 0
  • January 2021
  • PDF

This document was uploaded by user and they confirmed that they have the permission to share it. If you are author or own the copyright of this book, please report to us by using this DMCA report form. Report DMCA


Overview

Download & View Vision: Www.visionias.in as PDF for free.

More details

  • Words: 24,205
  • Pages: 42
Loading documents preview...
VISIONIAS www.visionias.in ANSWERS & EXPLANATIONS GENERAL STUDIES (P) TEST – 2982 (2020) Q 1.A 



Indian sugar industry is the second largest agro-based industry in India. The basic raw material is sugar cane, which has some specific qualities: o It is a weight losing raw material.It cannot be stored for long, because in that case, it loses the sucrose content.It can not be transported over long distances. Sugar factories hence are located within the cane producing regions. Hence statement 1 is correct. India is the second largest producer of both sugarcane and cane sugar and contributes about 8 per cent of the total sugar production in the world. An annual produce of 341,400 Thousand Metric Tons(TMT) was estimated in the year 2015. Brazil tops the list of sugarcane producers, with an annual production of 739,300 thousand metric tons. Besides, khandasari and gur or jaggery are also prepared from sugarcane. Hence statement 2 is not correct.

Q 2.A 

Orang National Park Orang National Park or Rajiv Gandhi Orang National Park, which is also known as the mini Kaziranga National Park, is located in the Darrang and Sonitpur districts of the state of Assam at a distance of approx 100 km from the Kaziranga National Park. o The Orang National Park is sprawl over an area of 78.81 square kilometers, lies on the north bank of the River Brahmaputra. Hence pair 1 is correctly matched. o The whole area of the park is divided into Water body, Eastern Himalayan Moist Deciduous Forest, Eastern Wet Alluvial Grassland, Eastern Seasonal Swamp Forest, Savannah Grasslands, Degraded Grassland, Moist Sandy area and Dry Sandy area. Panna National Park o Panna National Park is a national park located in Panna and Chhatarpur districts of Madhya Pradesh in India. It was declared as a National Park in the year of 1981. The park is known worldwide for its wildlife including tigers, deer, antelope, vultures, wolf, Chinkara, Cheetal and lots more. o Ken River flows through this reserve and creates beautiful waterfalls on its way to the valley. The biodiversity in this national park is extremely rich. Hence pair 2 is not correctly matched. o The vegetation of this region is mixed with uneven terrain with scrubby vegetation and grass, rocky landscape gathered along with hundreds verities of trees and shrubs. Jim corbett National Park o Jim Corbett National Park is the oldest national park in India and was established in 1936 as Hailey National Park to protect the endangered Bengal tiger. o It is located in Nainital district and Pauri Garhwal district of Uttarakhand and was named after Jim Corbett, a well known hunter and naturalist. The park was the first to come under the Project Tiger initiative. o Other mammals inhabiting Jim Corbett National park includes deer species (Barking, Sambhar, Hogg and Chital), Sloth bears and Himalayan Black bears, Indian Grey Mongoose, otters, Elephants, Yellow Throated Martens, Ghoral (Goat-Antelopes), Indian Pangolins, and Langur and Rhesus Monkeys. o Ramganga River is the Lifeline for Corbett National Park. It is the largest of all the precious water bodies in the park. Hence pair 3 is correctly matched. o





1

www.visionias.in

©Vision IAS

FREE BOOKS, NOTES & VIDEOS FOR CIVILSERVICES

EBOOKS & MAGZINES

UPSC PRELIMS MATERIALS

USPC MAINS MATERIALS

VIDEO FOR CIVILSERVICES

DAILY NEWSAPERS

SECUREIAS

UPSC PRELIMS TESTSERIES

UPSC MAINS TESTSERIES

DELHI STUDENTS

CIVILSERVICES BOOKS

OPTIONAL SUBJECTS BOOKS, STATE PCS, SSC, BANKING TEST SERIES, VIDEOS & NOTES BOOKS, TESTS VIDEOS & NOTES 1.GEOGRAPHY 2.HISTORY

1.UPPSC 4.IBPS

3.MATHEMATICS 4. SOCIOLOGY

ENGINEERING BOOKS & MATERIAL 1. IES 2. GATE 3. IFoS

5.PUBLIC ADMINISTRATION 6. POLITICAL SCIENCE

4. COMPUTER SCIENCE 5. MECHINICAL ENGINEERING

7. ECONOMICS

OTHER TELEGRAM CHANNELS

8 PHYSICS 9 COMMERCE ACCOUNTANCY 10 ANTHROPOLOGY 11 LAW 12 PHILOSOPHY 13 CHARTERED ACCOUNTANTANCY 14 MEDICAL SCIENCE

1 GOVERNMENT JOBS 2 LEARN YOGA & MEDITATION 3 LEARN ENGLISH 4 BEST DELAS & OFFERS 5 IAS HINDI BOOKS 6 PDFs FOR ALL EXAMS 7. WORLD DIGITAL LIBIRARY

1.CHENNAI STUDENTS 2.BANGLORE STUDENTS

2.SSC 3.MPSC 5.RAS & RPSC

3. CURRENT AFFAIRS

CONTACT FOR ADVERTISEMENT IN ABOVE CHANNLES ADMIN1:

ADMIN2:



Jaldapara National Park o Jaldapara National Park located in the Jalpaiguri district of the state of West Bengal. Jaldapara National Park also known as Jaldapara Wildlife Sanctuary. o It is home to a myriad variety of flora and fauna.Some of the prominent tree species found in the park include tall Sal and Shishu trees. Other variety of ferns, shrubs and tall grass can also be seen. o The main attraction in Jaldapara is the Indian one horned rhinoceros. The park is home to Indian one horned rhinoceros, Elephants, Indian Bison, Hog Deer, Spotted Deer, Chital, Sambar, Barking deer, Wild pigs, Tigers, Leopards, Jungle cat, Leopard cat, Fishing cat, Indian civet, Giant squirrel, Indian pangolin, Hispid hare, Indian porcupine etc. o The park is situated at the foothills of the Eastern Himalayas and on the bank of Torsa River. Hence pair 4 is not correctly matched.  The 358 km long Torsa River rises from the Chumbi Valley in Tibet, where it is known as Machu. It then flows into Bhutan, where it is known as the Amo Chu, before reaching West Bengal. About 113 km of the river is in China and 145 km in Bhutan.  In West Bengal, Torsa flows through the tea estate of Dalsingpara (in Jalpaiguri) and the Jaldapara National Park (Alipurduar District). It then flows into Bangladesh by the name of Kaljani and joins the Brahmaputra in Bangladesh.

Q 3.C 



Statement 1 is correct: Shifting cultivation is an agricultural system in which plots of land are cultivated temporarily, then abandoned until the soil loses its fertility) and allowed to revert to their natural vegetation while the cultivator moves on to another plot. Statement 2 is correct: Equatorial regions are very sparsely populated and in the forests of this region, the primitive people/tribes are usually hunters and collectors. Primitive subsistence agriculture or shifting cultivation is widely practised by many tribes in the tropics, especially in Africa, south and central America and south east Asia. It is prevalent in tropical region in different names, e.g. Jhuming in North eastern states of India, Milpa in Central America and Mexico and Ladang in Indonesia and Malaysia

Q 4.D 





Twenty-five species of marine mammals are found in Indian waters and belong to two orders Cetacea and Sirenia. Sea cow Dugong dugon occurs in the nearshore waters of Gulf of Mannar, Gulf of Kachchh and the Andaman and Nicobar Islands. One species belonging to the order Sirenia, namely the dugong, and 30-34 species of cetaceans (dolphins, whales, and porpoises) including the Ganges river dolphin, are found in the waters of the Indian subcontinent40. All species of marine mammals along the Indian coasts are protected under the Indian (Wildlife) Protection Act (1972). Porpoises are a group of fully aquatic marine mammals, all of which are classified under the family Phocoenidae. There are seven extant species of porpoise. They are small toothed whales that are very closely related to oceanic dolphins. Porpoise teeth are spade-shaped whilst dolphins are conical. A dolphin has a hooked or curved dorsal fin (except for those species that don't have a dorsal fin) whereas a porpoise has a more triangular dorsal fin, and generally speaking, dolphin bodies are leaner, although porpoises' are a little more chunky. Dolphin is a common name of aquatic mammals within the infraorder Cetacea. The dugong is a medium-sized marine mammal. It is one of four living species of the order Sirenia, which also includes three species of manatees. It is the only living representative of the once-diverse family Dugongidae; its closest modern relative, Steller's sea cow, was hunted to extinction in the 18th century.

Q 5.D 

2

Statement 1 is not correct: Ministry of Human Resource Development has announced a new PPP Scheme, National Educational Alliance for Technology (NEAT) for using technology for better learning outcomes in Higher Education. There are a number of start-up companies developing this and MHRD would like to recognise such efforts and bring them under a common platform so that learners can access it easily. Educating the youth is a National effort and MHRD proposes to create a National Alliance with such technology developing EdTech Companies through a PPP model. www.visionias.in

©Vision IAS



 

Statement 2 is correct: The objective is to use Artificial Intelligence to make learning more personalised and customised as per the requirements of the learner. This requires development of technologies in Adaptive Learning to address the diversity of learners. Statement 3 is not correct. AICTE would be the implementing agency for NEAT programme. The scheme shall be administered under the guidance of an Apex Committee constituted by MHRD. Hence, NOTE: National Knowledge Network (NKN) project is aimed at establishing a strong and robust Indian network which will be capable of providing secure and reliable connectivity.NKN is a state-of-the-art multi-gigabit pan-Indian resource-sharing network aimed at digitally connecting all national universities, colleges and research establishments to create ―country-wide virtual classrooms.

Q 6.A 



   

For the first time, ‗Geochemical Baseline Atlas of India‘ developed by CSIR-National Geophysical Research Institute (NGRI) for use by policy makers to assess environmental damage was released. Hence statement 2 is not correct. The atlas consisting 45 maps of metals, oxides and elements present in top and bottom soils across India will serve as a reference against which future generations of the country would be able to assess the chemical compositional changes on Earth‘s surface. These maps help in finding out future contamination caused by industries or other bodies which cause pollution. Hence statement 1 is correct. It will help the country to assess the presence of toxic chemicals in soil and how it varies from place to place. It also helps in identifying how soil pollution is contributing to groundwater contamination Further, the map will also support in choosing the right soil that‘s rich in certain micronutrient elements required for each plant growth. Earlier, there was no way to prove if polluters denied causing damage to environment. Now, the baseline maps atlas helps show evidence of it. CSIR- NGRI: The National Geophysical Research Institute (NGRI) is a geoscientific research organization established in 1961 under the Council of Scientific and Industrial Research (CSIR).

Q 7.D 







Aerosols are minute particles suspended in the atmosphere. When these particles are sufficiently large, we notice their presence as they scatter and absorb sunlight. Their scattering of sunlight can reduce visibility (haze) and redden sunrises and sunsets. Aerosols interact both directly and indirectly with the Earth's radiation budget and climate. As a direct effect, the aerosols scatter sunlight directly back into space. As an indirect effect, aerosols in the lower atmosphere can modify the size of cloud particles, changing how the clouds reflect and absorb sunlight, thereby affecting the Earth's energy budget. Aerosols also can act as sites for chemical reactions to take place (heterogeneous chemistry). The most significant of these reactions are those that lead to the destruction of stratospheric ozone. During winter in the polar regions, aerosols grow to form polar stratospheric clouds. The large surface areas of these cloud particles provide sites for chemical reactions to take place. These reactions lead to the formation of large amounts of reactive chlorine and, ultimately, to the destruction of ozone in the stratosphere. Hence all the statements are correct.

Q 8.C 



3

Statement 1 is not correct: The word smog is derived from two words- Smoke and Fog. There are two types of smog viz. Classical smog and Photochemical smog. Classical smog occurs in cool humid climate. It is a mixture of smoke, fog and sulphur dioxide. Chemically it is a reducing mixture and so it is also called as reducing smog. Statement 2 is not correct and statement 3 is correct: Photochemical smog occurs in warm, dry and sunny climate. The main components of the photochemical smog result from the action of sunlight on unsaturated hydrocarbons and nitrogen oxides produced by automobiles and factories. Photochemical smog has high concentration of oxidizing agents and is, therefore, called as oxidising smog. www.visionias.in

©Vision IAS

Q 9.C 



The Kayaks or backwaters of Kerala are a network of brackish lagoons and lakes lying parallel to the Arabian Sea coast (Malabar Coast) as well as interconnected canals, rivers, and inlets. Hence statement (c) is the correct answer. They are used for fishing, inland navigation and also due to its special attraction for tourists. Every year the famous Nehru Trophy Vallamkali (boat race) is held in Punnamada Kayal in Kerala. Backwaters in Kerala are a network of 1500 km of canals both manmade and natural, 38 rivers and 5 big lakes extending from one end of Kerala to the other. Backwaters are one of the major tourism product of Kerala, being unique to the state. Traditionally used as one of the main transportation alleys, today backwaters offer a rejuvenating experience for tourists visiting Kerala.

Q 10.B 





 

Recently, Kittur Utsava was organised to celebrate Queen Rani Channamma‘s victory over East India‘s company. The festival organized sports, cultural programmes and lectures by resource persons on the kingdom of Rani Channamma. Rani Chennamma, the queen of Kitturu was a warrior who led a war against British forces in the early part of 19th Century. She was among the first Indian rulers to lead an armed rebellion against the British East India Company in 1824. She was outnumbered and arrested, but she is still remembered for leading the revolt against British rule in India. She was married to Mallasarja Desai, the king of Kittur, at the age of 15 and became the queen of Kittur. She had one son from the marriage, who after the death of her husband in 1816, also died in 1824. As the queen of Kittur, Kittur Chennamma adopted Shivalingappa after the death of her only son with the aim of making him the heir to the throne of Kittur. The British East India Company did not take lightly to Chennamma‘s act and ordered Shivalingappa‘s exile from the kingdom under the pretext of the Doctrine of Lapse. However, Rani Chennamma defied the British order to expel Shivalingappa from the throne. This led to the outbreak of war. Hence statement 1 is not correct. During the freedom movement, her brave resistance against the British forces became the theme of several inspirational plays, folk songs (Lavani) and stories. Rani Chennamma‘s first victory against the British forces is still honoured annually in October during the ‗Kittur Utsava‘, held in Kittur. Hence, statement 2 is correct.

Q 11.C 



Recently, the Government of India has decided to launch the Jal Abhayaranya Programme for rejuvenation of Himalayan Springs. For this springs in eight Himalayan States have been identified. Following are the features of this programme: o Inventorization of nearly 1 lakh springs with identification and compilation of information on drying springs. o Rejuvenation of identified springs to be taken up in the next five years targeting the most vulnerable districts in the first year. o Implementation in collaboration with the village panchayats, state governments particularly the state forest departments in all 12 Indian Himalayan Region (IHR) states. Hence statement (c) is the correct answer.

Q 12.D 





4

The air is set in motion due to the differences in atmospheric pressure.The air in motion is called wind. The windblows from high pressure to low pressure. The wind at the surface experiences friction. In addition, rotation of the earth also affects the wind movement. The force exerted by the rotation of the earth is known as the Coriolis force. Thus, the horizontal winds near the earth surface respond to the combined effect of three forces – the pressure gradient force, the frictional force and the Coriolis force. In addition, the gravitational force acts downward. Pressure Gradient Force: The differences in atmospheric pressure produces a force. The rate of change of pressure with respect to distance is the pressure gradient. The pressure gradient is strong where the isobars are close to each other and is weak where the isobars are apart. www.visionias.in ©Vision IAS

 

Frictional Force: It affects the speed of the wind. It is greatest at the surface and its influence generally extends upto an elevation of 1 - 3 km. Over the seasurface the friction is minimal. Coriolis Force: The rotation of the earth about its axis affects the direction of the wind. This force is called the Coriolis force after the French physicist who described it in 1844. It deflects the wind to the right direction in the northern hemisphere and to the left in the southern hemisphere. The deflection is more when the wind velocity is high. o The Coriolis force is directly proportional to the angle of latitude. It is maximum near the poles and is absent at the equator. o The Coriolis force acts perpendicular to the pressure gradient force. The pressure gradient force is perpendicular to an isobar. The higher the pressure gradient force, the more is the velocity of the wind and the larger is the deflection in the direction of wind. o As a result of these two forces operating perpendicular to each other, in the low-pressure areas the wind blows around it. o At the equator, the Coriolis force is zero and the wind blows perpendicular to the isobars. The low pressure gets filled instead of getting intensified. That is the reason why tropical cyclones are not formed near the equator.

Q 13.A 





The Uyghurs are recognized as native to the Xinjiang Uyghur Autonomous Region of the People's Republic of China. They are considered to be one of China's 55 officially recognized ethnic minorities. The Uyghurs are recognized by China only as a regional minority within a multicultural nation and China rejects the idea of them being an indigenous group. Hence pair 1 is not correctly matched. The Rohingya people are a stateless Indo-Aryan ethnic group who predominantly follow Islam with a minority following Hinduism and reside in Rakhine State, Myanmar. There were an estimated 1 million Rohingya living in Myanmar before the 2016–17 crisis. Described by the United Nations in 2013 as one of the most persecuted minorities in the world, the Rohingya population is denied citizenship under the 1982 Myanmar nationality law. Hence pair 2 is correctly matched. The Hajong are a tribal people native to the Indian subcontinent, notably in the northeast Indian states and Bangladesh. The majority of the Hajongs are settled in India. Hajongs are predominantly rice farmers. Hajong have the status of a Scheduled Tribe in India.The Hajong people are spread out across northeast India and Bangladesh with the majority of the population on the India side of the border. In India, Hajongs are found in both the Garo and Khasi Hills of Meghalaya, largely along the South-West Garo Hills District of Meghalaya and Bangladesh border. Hence pair 3 is not correctly matched.

Q 14.D  



Milwaukee Deep, also known as The Milwaukee Depth, is the deepest part of the Atlantic Ocean and is part of the Puerto Rico Trench. Hence, pair 1 is correctly matched. The Sunda Trench, also known as Java Trench, is an oceanic trench located in the Indian Ocean near Sumatra. It is formed where the Australian-Capricorn plates subduct under a part of the Eurasian Plate. Hence, pair 2 is correctly matched. The Challenger Deep is the deepest known point in the Earth's seabed hydrosphere. It is located in the Pacific Ocean, at the southern end of the Mariana Trench near the Mariana Islands group. Hence, pair 3 is correctly matched.

Q 15.D 

 

5

All the exogenic geomorphic processes are covered under a general term, denudation. The word ‗denude‘ means to strip off or to uncover. Weathering, mass wasting/movements, erosion and transportation are included in denudation. Weathering: Weathering is the process of decomposition of rocks into smaller particles. This decomposition is done via three weathering means, physical, chemical and biological weathering. Decomposition and disintegration of rocks into fragments is done by physical weathering, in chemical weathering this decomposition is due to the chemical change in the rock formation, and in biological weathering animals and plants are responsible for decomposition of rocks. www.visionias.in

©Vision IAS

   

Erosion: Erosion is the movement of rocks and weathered materials due to natural agents like rivers and glaciers. Transportation: Transportation is the removal of eroded debris to new positions. Deposition: It is dumping of debris in certain parts of the earth, where it may accumulate to form new rocks. All the above denudation processes are taking in place simultaneously in different part of the worldat different rates, much depending on the nature of the relief, the structure of the rock, the local climate and interference by man.

Q 16.C 





Recently, officials of the Forest Department and Cuddalore Marine Police retrieved 5.88 kg of ambergris (sometimes known as whale vomit) that washed had ashore. Ambergris, is a solid waxy substance originating in the intestine of the sperm whale (Physeter catodon). In Eastern cultures ambergris is used for medicines and potions and as a spice; in the West it was used to stabilize the scent of fine perfumes. Ambergris floats and washes ashore most frequently on the coasts of China, Japan, Africa, and the Americas and on tropical islands such as the Bahamas. Fresh ambergris is black and soft and has a disagreeable odour. When exposed to sun, air, and seawater, however, it hardens and fades to a light gray or yellow, developing a subtle and pleasant fragrance in the process. In India, the import and export of ambergris is banned and possession of the substance was punishable under the Customs Act. Sperm whales mainly prey on squids, and the undigested portions are eliminated through their vomit. Ambergris is mainly used in perfumes and cosmetics and fetches a handsome price based on its age. It has an unpleasant smell after it is secreted but soon acquires a sweet smell.

Q 17.C

6



The southwest monsoon splits into two branches i.e. the Arabian Sea Branch and the Bay of Bengal Branch near the southernmost end of the Indian Peninsula. Hence, it arrives in India in two branches: the Bay of Bengal branch and the Arabian Sea Branch.



It first originates in the Bay of Bengal causing rainfall over the plains of north India. Second is the Arabian Sea current of the southwest monsoon which brings rain to the west coast of India. The latter extends toward a low-pressure area over the Thar Desert and is roughly three times stronger than the Bay of Bengal branch. www.visionias.in

©Vision IAS







The monsoon winds originating over the Arabian Sea further split into three branches: o One branch is obstructed by the Western Ghats. These winds climb the slopes of the Western Ghats and as a result of orographic rainfall phenomenon, the windward side of Ghats receives very heavy rainfall ranging between 250 cm and 400 cm. After crossing the Western Ghats, these winds descend and get heated up. This reduces humidity in the winds. As a result, these winds cause little rainfall east of the Western Ghats. This region of low rainfall is known as the rain-shadow area. o Another branch of the Arabian Sea monsoon strikes the coast north of Mumbai. Moving along the Narmada and Tapi river valleys, these winds cause rainfall in extensive areas of central India. The Chotanagpur plateau gets 15 cm rainfall from this part of the branch. Thereafter, they enter the Ganga plains and mingle with the Bay of Bengal branch. o A third branch of this monsoon wind strikes the Saurashtra Peninsula and the Kutch. It then passes over west Rajasthan and along the Aravallis, causing only a scanty rainfall. In Punjab and Haryana, it too joins the Bay of Bengal branch. These two branches, reinforced by each other, cause rains in the western Himalayas. Arabian Sea branch of the monsoon is much powerful than the bay of Bengal branch for two reasons: o The Arabian Sea is larger than the Bay of Bengal. o The entire Arabian sea current advances towards India whereas only a part of Bay of Bengal current enters India, the remainder proceeding to Myanmar, Thailand and Malaysia. Hence statement 1 is correct. Tamil Nadu coast remain dry during this season due to two factors: o The Tamil Nadu coast is situated parallel to the Bay of Bengal branch of the south-west monsoon. o It lies in the rainshadow area of the Arabian sea branch of the south-west monsoon. Hence statement 2 is not correct. o





Tamil Nadu coast receives rainfall during the retreating monsoon or NorthEast Monsoon during winters. The monsoon rains occur in wet spells of few days duration at a time. The wet spells are interspersed with rainless interval known as ‗breaks‘. These breaks in rainfall are related to the cyclonic depressions mainly formed at the head of the Bay of Bengal, and their crossing into the mainland. Besides the frequency and intensity of these depressions, the passage followed by them determines the spatial distribution of rainfall. Hence statement 3 is correct.

Q 18.D 



Statement 1 is not correct: Cartosat-3 is an advanced Indian Earth Observation satellite built and developed by ISRO. It is a third-generation agile advanced satellite having high-resolution imaging capability. It has a panchromatic resolution of 0.25 metres making it the imaging satellite with the highest resolution in the world is a major improvement from the previous payloads in the Cartosat series. Statement 2 is not correct: The CARTOSAT-3, with an overall mass of over 1,600 kilograms (a nanosatellite weighs less than 10 kg), will address the increased user demand for large-scale urban planning, rural resource and infrastructure development, coastal land use and land cover. Cartosat-3 was injected into an orbit of 509 km at an inclination of 97.5 degrees to the equator. Its potential uses include weather mapping, cartography or army defence, and strategic applications.

Q 19.B  

 

7

Pollution of water originates from human activities. Through different paths, pollution reaches surface or groundwater. Non-point source pollution is pollution resulting from many diffuse sources, in direct contrast to point source pollution which results from a single source. Point sources are relatively easy to identify and monitor. Non point sources of pollution are those where a source of pollution cannot be easily identified and managed, e.g., agricultural run-off (from farm, animals and crop-lands), acid rain, storm-water drainage (from streets, parking lots and lawns), etc. Easily identified source or place of pollution is called as point source. e.g., municipal and industrial discharge pipes where pollutants enter the water- source. Hence option (b) is correct. www.visionias.in ©Vision IAS

Q 20.D 

 

Statement 1 is not correct: Inner Line Permit (ILP) is an official travel document to allow inward travel of an Indian citizen into a state that is protected under the ILP system for a limited period. It is obligatory for Indian citizens from outside those states to obtain a permit for entering into the protected state. The document is an effort by the government to regulate movement to certain areas located near the international border of India. This is an offshoot of the Bengal Eastern Frontier Regulations, 1873, which protected Crown's interest in the tea, oil and elephant trade by prohibiting "British subjects" from entering into these "Protected Areas". Every Indian citizen who is not a resident of those areas, requires an ILP. However, NRIs, PIO card holders and OCI card holders are not eligible for ILP. They will have to apply for a special permit called the Protected/Restricted area permit. Statement 2 is not correct: Inner Line Permit (ILP) is an official travel document issued by the concerned state governments. Statement 3 is not correct: The system is in force today in only four Northeastern states — Arunachal Pradesh, Nagaland and Mizoram and Manipur — and no Indian citizen can visit any of these states unless he or she belongs to that state, nor can he or she overstay beyond the period specified in the ILP. The inclusion of Manipur in Inner Line Permit was also announced on 10 December 2019 and is in force since 1 st January 2020.

Q 21.C 





Biological Oxygen Demand (BOD): o BOD is defined as the amount of oxygen demanded by the micro-organisms in the sewage for the decomposition of bio-degradable matter under aerobic conditions. This is the most commonly used parameter to determine the strength of municipal or organic quality of the water i.e. it measures biodegradable pollutants. o The standard BOD test determines the amount of oxygen required by the micro-organisms for the decomposition of the bio-degradable matter present in the wastewater sample under 5 days of aerobic condition at a temperature of 20 degree Celsius. It is measured in mg/l o BOD is determined for designing treatment facilities. It is used to determine the size of activated sludge units and trickling filter units. It is also used to determine the efficiency of various treatment units. Chemical Oxygen Demand (COD): o The amount of oxygen that is required for the chemical oxidation of the organic and inorganic chemicals present in the wastewater by utilising oxidising agents like Potassium permanganate, Potassium dichromate etc. is called as chemical oxygen demand (COD). o The presence of COD facilitates rapid chemical oxidation of organic matter without any additional equipment. This is the only method that enables to determine the organic load in heavy toxic sewage. Hence it measures both biodegradable pollutants and non-biodegradable pollutants. Key difference between BOD and COD: BOD measures the amount of oxygen required by the aerobic organisms to decompose organic matter and COD measures the oxygen required to decompose organic and inorganic constituents present in the wastewater by chemical reaction. Hence, the value of COD is greater than BOD.

Q 22.A 

8

The Cabinet Committee on Investment & Growth is headed by the Prime Minister and has four other members - Union Home minister, Union Highways and MSME minister, Union Finance minister, Union Commerce & Railways minister. Hence, statement 1 correct. It is entrusted with the following functions -: o To identify key projects required to be implemented on a time-bound basis, involving investments of Rs. 1000 crore or more, or any other critical projects; Hence, statement 2 is correct. o To prescribe time limits for issue of requisite approvals and clearances by the Ministries/Departments concerned in respect of projects in identified sectors; o To monitor the progress of identified projects including the time prescribed/taken to obtain each approval/clearance and delays, if any; o To review implementation of projects, that have been delayed beyond the stipulated timeframe, including issues causing delay in grant of clearances/approvals; www.visionias.in

©Vision IAS

o  

To review the procedures followed by Ministries/Departments to grant/refuse approvals and clearances. As per the Transaction of Business Rules, the Government constitutes the Cabinet Committees and decides on their functions. Hence, statement 1 is correct. The Cabinet Secretariat which functions directly under the Prime Minister acts as the secretariat to the Cabinet and Cabinet Committees. Hence, statement 3 is not correct.

Q 23.B Noctilucent Clouds  Noctilucent clouds, or night shining clouds, are tenuous cloud-like phenomena in the upper atmosphere of Earth. They consist of ice crystals and are only visible during astronomical twilight. Noctilucent roughly means "night shining" in Latin.  They are most often observed during the summer months from latitudes between 50° and 70° north and south of the Equator. Hence statement 2 is correct.  They are visible only during local summer months and when the Sun is below the observer's horizon, but while the clouds are still in sunlight.  Recent studies suggest that increased atmospheric methane emissions produce additional water vapor once the methane molecules reach the mesosphere – creating, or reinforcing existing noctilucent clouds.  They are the highest clouds in Earth's atmosphere.  They are located in the mesosphere at altitudes of around 76 to 85 km (47 to 53 mi). Hence statement 1 is not correct.  They are too faint to be seen in daylight, and are visible only when illuminated by sunlight from below the horizon while the lower layers of the atmosphere are in Earth's shadow. Q 24.B 

 



A biosimilar is a biologic medical producs (Biotherapeutics) highly similar to another already approved biological medicine (the 'reference medicine'). Biosimilars are approved according to the same standards of pharmaceutical quality, safety and efficacy that apply to all biological medicines.. Biosimilars are officially approved versions of original "innovator" products and can be manufactured when the original product's patent expires. Biotherapeutics are pharmaceutical products derived from biological and living sources. They include therapeutic vaccines, blood, blood components, cells, gene therapies, tissues and other materials. Biosimilars are often confused with generic medicines — but they are not exactly the same. o Generic medicines are chemically synthesized while biosimilars are grown in complex living systems. Hence statement (b) is not correct. o Biological medicines are large, complex molecules or mixtures of molecules that may be composed of living material — as such, biosimilars are unlikely to be exact copies of their reference product Recently, The World Health Organization (WHO) prequalified its first biosimilar medicine – trastuzumab – in a move that could make this expensive, life-saving treatment more affordable and available to women globally.Trastuzumab – a monoclonal antibody – was included in the WHO Essential Medicines List in 2015 as an essential treatment for about 20% of breast cancers. It has shown high efficacy in curing early stage breast cancer and in some cases more advanced forms of the disease. Biotherapeutic medicines, which are produced from biological sources such as cells rather than synthesized chemicals, are important treatments for some cancers and other non-communicable diseases. Like generic medicines, biosimilars can be much less expensive versions of innovator biotherapeutics while keeping the same effectiveness. They are usually manufactured by other companies once the patent on the original product has expired.

Q 25.A  

9

Albedo is the portion of solar energy reflected from the surface of the Earth back into space. It is a reflection coefficient and has a value less than one. When the solar radiation passes through the atmosphere, a certain amount of it is scattered, reflected and absorbed. The reflected sum of radiation is called as the albedo of the earth. www.visionias.in

©Vision IAS







Albedo is an important concept in climatology, astronomy, and environmental management. It plays a major role in the energy balance of the earth‘s surface, as it defines the rate of the absorbed portion of the incident solar radiation. The difference in the average albedo of Earth has an important influence on the temperature of the Earth. If the average albedo is lower than the previous year‘s albedo, it specifies that amount of radiation absorbed is higher . This results in the rise in the temperature of the Earth.Earth‘s albedo is constantly measured using satellites to monitor the global warming. Albedo of different kinds of surfaces: Surfaces Snow cover Sand Grass Dry Ground Wet Ground Grassland Water Thick clouds Thin clouds Black soil

Percentage of radiation being reflected 70-90 20-30 14-37 15-25 10 10-25 3-5 70-80 25-50 8-14

Q 26.B 

According to Plate Tectonics Theory, A tectonic plate (also called lithospheric plate) is a massive, irregularly-shaped slab of solid rock, generally composed of both continental and oceanic lithosphere. Plates move horizontally over the asthenosphere as rigid units. There are three types plate interaction, also called plate boundaries: o Divergent Boundaries are where new crust is generated as the plates pull away from each other. The sites where the plates move away from each other are called spreading sites. The best-known example of divergent boundaries is the Mid-Atlantic Ridge. Other examples are Red Sea Rift, Baikal Rift Zone East African Rift. Hence, pair 1 is correctly matched. o Convergent boundaries are locations where lithospheric plates are moving towards one another. The plate collisions that occur in these areas can produce earthquakes, volcanic activity, and crustal deformation. Japan, the Aleutian Islands, Himalayas etc. are some of the examples. Hence, pair 2 is correctly matched. o Transform Boundaries are where the crust is neither produced nor destroyed as the plates slide horizontally past each other. Transform faults are the planes of separation generally perpendicular to the midoceanic ridges. The San Andreas is a common examples for this type of interaction. Hence, pair 3 is correctly matched.

Q 27.A 



Chinese President Xi Jinping and his Russian counterpart Vladimir Putin inaugurated the ―Power of Siberia‖ gas pipeline — a massive cross-border undertaking not only central to China‘s energy security but also for bolstering special ties between Beijing and Moscow. The 30-year project is anchored by a $400 billion gas deal. It is a Gazprom-operated pipeline in Eastern Siberia that transports natural gas from Yakutia to Primorsky Krai and China. It is a part of the eastern gas route from Siberia to China

Q 28.C 

10

The New Wildlife Action Plan 2017-2031 (NWAP) sets out the framework for governmental intervention at a time when habitat loss and human-wildlife conflict are becoming increasingly commonplace in the country. The NWAP is the policy framework on which management plans for the protected areas www.visionias.in

©Vision IAS

 



(PAs) will be developed in the coming 15 years. The current plan calls for adopting a landscape approach for wildlife conservation which is an improvement over the previous protected areacentric approach. Hence both the statements are correct. The action plan launched recently is the third one for the country – the first was from 1983 to 2001, the second from 2002 to 2016. The NWAP states that taking the landscape approach to wildlife conservation can help in look at the issue of human-wildlife conflicts, which have been increasing in frequency in the recent years, in its broader context. While animals keep moving out of PAs, humans keep intruding into wildlife habitats. Thus the traditional approach of looking at conservation within the boundaries of PAs is not as relevant today, and hence the need to look at landscapes in their entirety so that development and conservation can be prioritised simultaneously. It also, for the first time, recognizes the concerns related to climate change impacts on wildlife, by integrating actions that need to be taken for their mitigation and adoption in wildlife management planning process. The thematic focus on climate change is also a new approach. Accepting climate change as a reality, the NWAP plans to promote research specific to climate change, including long-term monitoring and assessment of change in the distribution of vegetation types and ecosystems. Also, there is an emphasis on vulnerability mapping for fires, epidemics, drought and other environmental stresses that come with the changing climate.

Q 29.A 







Statement 1 is not correct: The National Green Tribunal has been established on 18.10.2010 under the National Green Tribunal Act 2010 for effective and expeditious disposal of cases relating to environmental protection and conservation of forests and other natural resources including enforcement of any legal right relating to environment and giving relief and compensation for damages to persons and property and for matters connected therewith or incidental thereto. Statement 2 is correct: The National Green Tribunal Act, 2010 under Section 19 gives the Tribunal power to regulate its own procedure. Additionally, the Tribunal is not bound by procedure under the Code of Civil Procedure, 1908 or the Indian Evidence Act, 1872 and is guided by principles of natural justice. However, the Tribunal is vested with the powers of a civil court under the Code of Civil Procedure for discharging its functions. The Tribunal has framed its own rules since applications to the tribunal are inherently distinct from civil suits or writ petitions. The Tribunal identifies necessary parties as required under necessary statutes mentioned in Schedule I of the NGT Act, 2010 and requires them to promptly respond by email which saves time and cost. Notice is not issued mechanically to every party named as respondents in the application. Statement 3 is correct: The Tribunal is mandated to make and endeavour for disposal of applications or appeals finally within 6 months of filing of the same. Initially, the NGT is proposed to be set up at five places of sittings and will follow circuit procedure for making itself more accessible. New Delhi is the Principal Place of Sitting of the Tribunal and Bhopal, Pune, Kolkata and Chennai shall be the other four place of sitting of the Tribunal.

Q 30.C 





11

Climate Change and Land is an IPCC Special Report on climate change, desertification, land degradation, sustainable land management, food security, and greenhouse gas fluxes in terrestrial ecosystems. It was released in August 2019. Special Report on the Ocean and Cryosphere in a Changing Climate: This Special Report on the Ocean and Cryosphere in a Changing Climate (SROCC) was prepared following an IPCC Panel decision in 2016 to prepare three Special Reports during the Sixth Assessment Cycle. The SROCC follows the other two Special Reports on Global Warming of 1.5°C (SR1.5) and on Climate Change and Land (SRCCL). It was released in September 2019. Global Warming of 1.5 ºC is an IPCC special report on the impacts of global warming of 1.5 °C above pre-industrial levels and related global greenhouse gas emission pathways, in the context of strengthening www.visionias.in ©Vision IAS

the global response to the threat of climate change, sustainable development, and efforts to eradicate poverty. It was released in October 2018. Q 31.D 











Incineration of plastic waste in an open field is a major source of air pollution. Most of the times, the municipal solid waste containing about 12% of plastics is burnt, releasing toxic gases like Dioxins, Furans, Mercury and Polychlorinated Biphenyls into the atmosphere. Dioxin is a general term that describes a group of hundreds of chemicals that are highly persistent in the environment. The most toxic compound is 2,3,7,8-tetrachlorodibenzo-p-dioxin or TCDD. o Dioxin is formed as an unintentional by-product of many industrial processes involving chlorine such as waste incineration, chemical and pesticide manufacturing and pulp and paper bleaching. Dioxin was the primary toxic component of Agent Orange (Agent Orange is a herbicide and defoliant chemical, one of the "tactical use" Rainbow Herbicides. It is widely known for its use by the U.S. military as part of its chemical warfare program, Operation Ranch Hand, during the Vietnam War from 1961 to 1971.) Furan is a heterocyclic organic compound, it is a colorless, flammable, highly volatile liquid with a boiling point close to room temperature. It is soluble in common organic solvents, including alcohol, ether, and acetone, and is slightly soluble in water. Lead: It is present in petrol, diesel, lead batteries, paints, hair dye products, etc. It can cause nervous system damage and digestive problems and, in some cases, cause cancer. Lead affects children in particular. Mercury: Although a comparatively rare element, mercury is ubiquitous in the environment, the result of natural geological activity and man-made pollution. Mercury from natural sources can enter the aquatic environment via weathering, dissolution and biological processes. Although extremely useful to man, mercury is also highly toxic to the human organism, especially in the form of methyl mercury, because it cannot be excreted and therefore acts as a cumulative poison. Barium: It is a silvery-white metal that takes on a silver-yellow color when exposed to air. Barium occurs in nature in many different forms called compounds. These compounds are solids, existing as powders or crystals, and they do not burn well.

Q 32.A 











Mechanical disruption and reorganisation of the original minerals within rocks due to breaking and crushing without any appreciable chemical changes is called dynamic metamorphism. The materials of rocks chemically alter and recrystallise due to thermal metamorphism. There are two types of thermal metamorphism — Contact metamorphism and regional metamorphism. In contact metamorphism, the rocks come in contact with hot intruding magma and lava and the rock materials recrystallise under high temperatures. Quite often new materials form out of magma or lava are added to the rocks. In regional metamorphism, rocks undergo recrystallisation due to deformation caused by tectonic shearing together with high temperature or pressure or both. In the process of metamorphism in some rocks grains or minerals get arranged in layers or lines. Such an arrangement of minerals or grains in metamorphic rocks is called foliation or lineation. Exfoliation is the term used to describe the peeling away of sheets of rock millimetres to meters in thickness from a rock's surface due a range of physical and chemical processes during exhumation and weathering. Mineralization (in soil science) is the decomposition of the chemical compounds in organic matter, by which the nutrients in those compounds are released in soluble inorganic forms that may be available to plants. Lithification is the process in which sediments compact under pressure, expel connate fluids, and gradually become solid rock.

Q 33.A 

12

Deltas are depositional landforms. The load carried by the rivers is dumped and spread into the sea. If this load is not carried away far into the sea or distributed along the coast, it spreads and accumulates as a low cone, called as Delta. Conditions favourable for the formation of delta are: www.visionias.in

©Vision IAS

o o o o o

Active vertical and lateral erosion in the upper course of the river to provide extensive sediments to be eventually deposited as deltas. Hence, statement 1 is correct. The coast should be sheltered preferably tideless. The sea adjoining the delta should be shallow or else the load will disappear in the deep waters. Hence, statement 2 is not correct. There should be no large lakes in the river course to 'filter off the sediments. Hence, statement 3 is not correct. There should be no strong current running at right angles to the river mouth, washing away the sediments.

Q 34.C 





The Make in India Mittelstand (MIIM) was launched to encourage German small and medium industry (SME) to India has evoked major interest in Germany, giving further boost to bilateral business ties. It is a Market Entry Support Programme for Facilitating Investments by German Mittelstand and Family Owned Companies in India. The programme is being implemented with the support of not only the concerned Central and State Government Ministries in India but also by engaging key industry partners who can support the companies in various aspects of market entry into India. The recent Indo German meet, welcomed the success of the MIIM programme which has facilitated over 135 german mittelstand companies with a declared investment of more than 1.25 Billion Euros.

Q 35.A 





Joint Forest Management ( JFM) is an approach and program initiated in the context of the National Forest Policy of 1988 wherein state forest departments support local forest-dwelling and forest fringe communities to protect and manage forests and share the costs and benefits from the forests with them. Communities organize themselves into a JFM Committee to protect and manage nearby forests, guided by locally prepared bye-laws and micro plans. The key element in JFM is that communities have the power to manage the use of forests by members and also exclude non-members. A Joint Forest Management Committee ( or JFMC in short) is a democratic, decentralized and transparent local institution of forest and forest fringe dwelling communities, that is part of the Gram Sabha fully or partially and set up as per the provisions of applicable JFM rules/guidelines of the state. Typically, there is one JFMC in one revenue village. An Eco-Development Committee (EDC) is similar to JFMCs but meant for villages in Protected Areas and their buffer zones. Their setup, working, role, responsibilities, powers, funds etc. are as per the state-level orders. Hence statement 1 is correct and statement 2 is not correct. More than 1,18,213 JFMCs involving around 20 million people are managing over 25 million ha of forest area. Such-management promises to convert low productivity and poorly stocked forests into rich forests which in turn increase carbon stocks. JFM covers approximately29.8% of the total forest area of the country (REDD+ Strategy India, 2018)

Q 36.B 



13

Earth‘s magnetic field is defined by the North and South Poles that align generally with the axis of rotation. The lines of magnetic force flow into Earth in the northern hemisphere and out of Earth in the southern hemisphere. In Earth, heat is being transferred from the solid inner core to the liquid outer core, and this leads to convection of the liquid iron of the outer core. Because iron is a metal and conducts electricity (even when molten), its motion generates a magnetic field. So, Earth‘s magnetic field is generated within the outer core (not the inner core) by the convective movement of liquid iron. Hence, statement 1 is not correct. The magnetic field of Earth is not stable over geological time. For reasons that are not completely understood, the magnetic field decays periodically and then becomes re-established. When it does rewww.visionias.in

©Vision IAS

establish, it may be oriented the way it was before the decay, or it may be oriented with the reversed polarity. These changes in Earth‘s magnetic field can also be used to date events in geologic history. Hence, statement 2 is correct. Q 37.B 



 



Statement 1 is correct: Extra tropical cyclones form along the polar front. In the northern hemisphere, warm air blows from the south and cold air from the north of the front. When the pressure drops along the front, the warm air moves northwards and the cold air move towards, south setting in motion an anticlockwise cyclonic circulation. The cyclonic circulation leads to a well-developed extra tropical cyclone, with a warm front and a cold front. The extra tropical cyclone differs from the tropical cyclone in number of ways: o The extra-tropical cyclones have a clear frontal system which is not present in the tropical cyclones. o They cover a larger area and can originate over the land and sea, whereas the tropical cyclones originate only over the seas and on reaching the land they dissipate. o The extra-tropical cyclone affects a much larger area as compared to the tropical cyclone. o The wind velocity in a tropical cyclone is much higher and it is more destructive. Statement 2 is not correct: The extra tropical cyclones move from west to east but tropical cyclones, move from east to west. Tropical cyclones are violent storms that originate over oceans in tropical areas and move over to the coastal areas bringing about large-scale destruction caused by violent winds, very heavy rainfall and storm surges. Statement 3 is correct: The conditions favourable for the formation and intensification of tropical storms are: o Large sea surface with temperature higher than 27° C o Presence of the Coriolis force (hence statement 3 is correct) o Small variations in the vertical wind speed o A pre-existing weak low-pressure area or low-level-cyclonic circulation o Upper divergence above the sea level system.

Q 38.B   





The Human Development Report (HDR) is an annual report published by the Human Development Report Office of the United Nations Development Programme (UNDP). India ranks 130 among 189 countries in the latest human development index released. Within South Asia, India's human development index (HDI) value is above the average of 0.638 for the region The report was first launched in 1990 by the Pakistani economist Mahbub ul Haq and Indian Nobel laureate Amartya Sen. Its goal was to place people at the center of the development process in terms of economic debate, policy and advocacy. The first HDR in 1990 shifted the economic growth-centric paradigm to the one that also includes giving people opportunities and freedoms to live the lives they would value. The Human Development Index (HDI) published in the report is a statistic composite index of life expectancy, education, and per capita income indicators, which are used to rank countries into four tiers of human development. The 2010 Human Development Report introduced an Inequality-adjusted Human Development Index (IHDI).

Q 39.B 



14

Recently, the Government of India has identified 100 major wetlands across the country for restoration and rejuvenation. Preparation of Integrated Management Plans for all wetlands has been initiated. The country's space agency ISRO had in 2011 come out with a national wetlands atlas on the basis of satellite images, mapping over 2 lakh wetlands covering around 4.63% of the total geographic area of India. The wetlands are actually land areas covered by water, either temporarily/seasonally or permanently. Marsh, fen and peatland come under this category. Hence statement 1 is correct.

www.visionias.in

©Vision IAS

FREE BOOKS, NOTES & VIDEOS FOR CIVILSERVICES

EBOOKS & MAGZINES

UPSC PRELIMS MATERIALS

USPC MAINS MATERIALS

VIDEO FOR CIVILSERVICES

DAILY NEWSAPERS

SECUREIAS

UPSC PRELIMS TESTSERIES

UPSC MAINS TESTSERIES

DELHI STUDENTS

CIVILSERVICES BOOKS

OPTIONAL SUBJECTS BOOKS, STATE PCS, SSC, BANKING TEST SERIES, VIDEOS & NOTES BOOKS, TESTS VIDEOS & NOTES 1.GEOGRAPHY 2.HISTORY

1.UPPSC 4.IBPS

3.MATHEMATICS 4. SOCIOLOGY

ENGINEERING BOOKS & MATERIAL 1. IES 2. GATE 3. IFoS

5.PUBLIC ADMINISTRATION 6. POLITICAL SCIENCE

4. COMPUTER SCIENCE 5. MECHINICAL ENGINEERING

7. ECONOMICS

OTHER TELEGRAM CHANNELS

8 PHYSICS 9 COMMERCE ACCOUNTANCY 10 ANTHROPOLOGY 11 LAW 12 PHILOSOPHY 13 CHARTERED ACCOUNTANTANCY 14 MEDICAL SCIENCE

1 GOVERNMENT JOBS 2 LEARN YOGA & MEDITATION 3 LEARN ENGLISH 4 BEST DELAS & OFFERS 5 IAS HINDI BOOKS 6 PDFs FOR ALL EXAMS 7. WORLD DIGITAL LIBIRARY

1.CHENNAI STUDENTS 2.BANGLORE STUDENTS

2.SSC 3.MPSC 5.RAS & RPSC

3. CURRENT AFFAIRS

CONTACT FOR ADVERTISEMENT IN ABOVE CHANNLES ADMIN1:

ADMIN2:





The government has also realized that community engagement is the key to conserve wetlands and other water bodies. The ministry has therefore decided to form 'Wetland Mitras', a group of self-motivated individuals for taking care of the identified wetlands across the country. Hence statement 2 is not correct. Statement 3 is correct: A concept of 'Wetland Health Card' has also been introduced to monitor the health of wetlands based on multiple parameters.

Q 40.D 





Pair 1 is not correctly matched: Ripu and Chirang are two reserve forests with contiguous habitat, in extreme western Assam bordering West Bengal and Bhutan. They are also part of the buffer zone of Manas Tiger Reserve. Ripu and Chirang are vital habitats that serve as a corridor linking Manas National Park, Assam with Buxa in West Bengal, and also the forests of Bhutan, especially important for Asian Elephants Elaphas maximus. It is a MIKE site in India. Pair 2 is not correctly matched: Dipor Bil, also spelt Deepor Beel, is located to the south-west of Guwahati city, in Kamrup district of Assam, India It is a permanent freshwater lake, in a former channel of the Brahmaputra River, to the south of the main river. It is a Wildlife Sanctuary of the Government of Assam and an important Ramsar site since 2002. Pair 3 is correctly matched: Kaziranga, located in the Northeastern state of Assam in the flood plains of the Brahmaputra River's south bank, was declared a World Heritage Site by UNESCO in 1985 for its unique natural environment. It was first established as a reserved forest in 1908 to protect the dwindling numbers of rhinoceros.

Q 41.D      

Saline Soils are also known as Usara soils. Saline soils contain a larger proportion of sodium, potassium, and magnesium, and thus, they are infertile and do not support any vegetative growth. They have more salts, largely because of the dry climate and poor drainage. They occur in arid and semiarid regions, and in waterlogged and swampy areas. Their structure ranges from sandy to loamy. They lack nitrogen and calcium. Saline soils are more widespread in western Gujarat, deltas of the eastern coast and in Sunderban areas of West Bengal. In the Rann of Kuchchh, the Southwest Monsoon brings salt particles and deposits there as a crust. Hence option 1 is correct. Seawater intrusions in the deltas promote the occurrence of saline soils. Hence option 2 is correct. In the areas of intensive cultivation with excessive use of irrigation, especially in areas of the green revolution, the fertile alluvial soils are becoming saline. Excessive irrigation with dry climatic conditions promotes capillary action, which results in the deposition of salt on the top layer of the soil. In such areas, especially in Punjab and Haryana, farmers are advised to add gypsum to solve the problem of salinity in the soil. Hence option 3 is correct.

Q 42.B 





15

India is one of the first countries in the world to develop a comprehensive Cooling Action plan which has a long term vision to address the cooling requirement across sectors and lists out actions which can help reduce the cooling demand. The India Cooling Action seeks to: o reduce cooling demand across sectors by 20% to 25% by 2037-38, o reduce refrigerants demand by 25% to 30% by 2037-38. Hence statement 1 is not correct. o reduce cooling energy requirements by 25% to 40% by 2037-38, o recognize ―cooling and related areas‖ as a thrust area of research under the national S&T Programme, o training and certification of 100,000 servicing sector technicians by 2022-23, synergizing with Skill India Mission. Hence statement 2 is correct. The thrust of the India Cooling Action Plan (ICAP) is to look for synergies in actions for securing both environmental and socio-economic benefits. ―The overarching goal of ICAP is to provide sustainable cooling and thermal comfort for all while securing environmental and socio-economic benefits for the society. This will also help in reducing both direct and indirect emissions. These actions will have significant climate benefits. The following benefits would accrue to the society over and above the environmental benefits: o Thermal comfort for all – provision for cooling for EWS and LIG housing o Sustainable cooling – low GHG emissions related to cooling www.visionias.in

©Vision IAS

o o o o

Doubling Farmers Income – better cold chain infrastructure – better value of produce to farmers, less wastage of produce Skilled workforce for better livelihoods and environmental protection Make in India – domestic manufacturing of air-conditioning and related cooling equipment‘s Robust R&D on alternative cooling technologies – to provide push to innovation in cooling sector.

Q 43.C 



Statement 1 is correct: The National Adaptation Fund for Climate Change (NAFCC) is a Central Sector Scheme which was set up in the year 2015-16. The overall aim of NAFCC is to support concrete adaptation activities which mitigate the adverse effects of climate change. The activities under this scheme are implemented in a project mode. The projects related to adaptation in sectors such as agriculture, animal husbandry, water, forestry, tourism etc. are eligible for funding under NAFCC. Statement 2 is correct: Considering the existing arrangement with NABARD as National Implementing Entity (NIE) for Adaptation Fund (AF) under Kyoto Protocol and its presence across the country, NABARD has been designated as National Implementing Entity (NIE) for implementation of adaptation projects under NAFCC. Under this arrangement, NABARD aids in facilitating identification of project ideas/concepts from State Action Plan for Climate Change (SAPCC), project formulation, appraisal, sanction, disbursement of fund, monitoring & evaluation and capacity building of stakeholders including State Governments.

Q 44.B 







A first-of-its-kind rehabilitation centre for freshwater turtles will be inaugurated in Bihar‘s Bhagalpur forest division in January 2020. The rehab centre, spread over half a hectare, will be able to shelter 500 turtles at a time. According to a recent study conducted by Traffic India, around 11,000 turtles are being smuggled in India every year. Statement 1 is correct: According to environmentalists, the turtles play a significant role in the river by scavenging dead organic materials and diseased fish, controlling fish population as predators and controlling aquatic plants and weeds. They are also described as indicators of healthy aquatic ecosystems. However, these species are now under severe threats due to habitat fragmentation and loss through dams and barrages, pollution, illegal poaching, accidental drowning through fishing nets and threats to their nesting habitats. Statement 3 is correct: The turtles have come under serious threat primarily for two reasons — food and the flourishing pet trade. Turtles are being frequently targeted for meat due to the prevailing belief that it gives an energy boost and keeps various diseases away. Usually, soft-shell turtles are victims of this belief. On the other hand, hard-shell turtles, especially spotted ones, are being poached for the pet trade. Such turtles are in high demand in south-east Asia, China and Japan. Statement 2 is not correct: India harbours 24 species of freshwater turtles and 4 species of tortoises, with areas in Northeast and North India considered in the top three Turtle Biodiversity Hotspots of the world. There are only five species of sea turtles are known to inhabit Indian coastal waters and islands. These are the Olive Ridley (Lepidochelys olivacea), Green (Chelonia mydas), Hawksbill (Eretmochelys imbricata), Loggerhead (Caretta caretta) and the Leatherback (Dermochelys coriacea) turtles.

Q 45.D 

A desert is a barren area of landscape where little precipitation occurs and consequently, living conditions are hostile to plant and animal life. In other words, it is an extremely dry area of land with sparse vegetation  The following are the reasons for the presence of hot and tropical desert on the western part of the continents: Presence of tropical easterly.  Most of the world‘s deserts are located in the western margins of continents in the subtropics because the prevailing winds in the tropics are tropical easterly winds.  When the moisture-laden trade wind flows from east to west shed their moisture on the eastern part and the tropical easterly winds become dry by the time they reach the western margins of the continents and so 16

www.visionias.in

©Vision IAS

they bring no rainfall. Thus, the region becomes devoid of moisture which causes dry conditions leading to the formation of deserts. Anticyclonic conditions(zone of horse latitude):  The areas between 20 ° -30 ° latitude on western margins of continents are the regions of descending air. Because of this, the air gets compressed and warm as it descends and thus the moisture keeps decreasing  Formation of Rain-shadow Zone:  A region in the lee of mountains that receives less rainfall than the region windward of the mountains is called a rain-shadow zone. For example Thar Desert in India is formed due to the formation of rainshadow zone because Aravallis mountains are situated parallel to the region. Therefore the moistureholding winds pass away from the region because there is an absence of mountain barriers. Presence of cold ocean currents  Presence of cold ocean currents along the western coast of continents tends to stabilize the air over the coast. This prevents cloud formation and rainfall. Hence, it leads to arid conditions or the formation of marine deserts on the adjacent coastal lands.  Examples Sonoran Desert of North America, the Peru and Atacama deserts of South America, and the Sahara [Moroccan part] and Namib deserts of Africa). Q 46.D 

Ministry of Women and Child Development along with Bill & Melinda Gates Foundation (BMGF) has recently announced the setting up of Bharatiya Poshan Krishi Kosh (BPKK). It would be a repository of diverse crops across 128 agro-climatic zones in India for better nutritional outcomes.Bhartiya Poshan Krishi Kosh‖ project has two components -: o Development of a Food Atlas  The Agro-Food Atlas is to act as a repository of diverse crops across 127 agro-climatic zones of the country having three parts- crops currently being grown, agro-ecological conditions (soil, organic carbon content, ground water availability etc) and guidance on how a greater diversity of crops could be encouraged in a particular district or block to promote dietary diversity and nutrition. o Documentation of promising practices for Jan-Andolan for POSHAN Abhiyaan.  It documents social, behavioral and cultural practices that promote and reinforce healthy dietary behaviors. Identification of promising practices with the help of a multi-disciplinary group of experts and developments of a tool kit to disseminate best strategies for Social and Behavioural Change Communication, specific to population groups in those regions are also a part of it. o Hence, option (d) is the correct answer.

Q 47.A  



The Aravalli Range is one of the most popular mountain ranges and the oldest fold mountain ranges in the world which stretching about 300 miles from the northeast to the southwest. The Aravallis mountain range streches from gujarat in the southeast to Delhi in the northeast for about 700 km and sculpts the topography of eastern Rajasthan and adjoining parts of Haryana and Delhi, creating a picturesque backdrop for flourishing settlements, with the highest point at Gurushikhar on Mount Abu in Rajasthan. Hence statements 1 and 2 are not correct. Features of Aravalli Mountain Range: o It is dry deciduous forest o Very rich in floral diversity especially medicinal plants like Kadaya (giving medicinal gum), Gugal, amla, Moosli, Khair, Salai, Modad, Dhavada, Khakhara, Timru are rare flora of this region. o The Aravalli range is very rich in natural resources and gave rise of numerous peninsula rivers like Banas, Luni, Sakhi, and Sabarmati. o Malwa plateau lies between Aravali and Vindhyan ranges. Hence statement 3 is correct.

Q 48.C 

17

Statement 1 is correct: Aestivation or æstivation is a state of animal dormancy, similar to hibernation, although taking place in the summer rather than the winter. Aestivation is characterized by inactivity and a lowered metabolic rate, that is entered in response to high temperatures and arid conditions. www.visionias.in

©Vision IAS





Statement 2 is not correct: Behavioural adaptations are observed in both animals and plants. Plant adaptations can be structural, behavioural or physiological. Behavioural adaptations of plants are behaviours which give them an advantage. All plant shoots grow quickly towards the light to maximise photosynthesis. Growth towards the light and other tropisms ensure that plants can respond to changes in their environment. Plant roots which grow downwards may be because of gravity or growing directly towards water to maximise photosynthesis. Other plants like the Venus flytrap have evolved structural and behavioural adaptations to catch insects. The flytrap itself is a structural adaptation and the closing of the trap to catch an insect is a behavioural adaptation. Statement 3 is correct: Aquatic plants show adaptations. For example, the seaweed is an aquatic plant. It is adapted for underwater life. This plant has its own air bubble in each leaf that provides the necessary space for the exchange of oxygen from the water to the plant. It also helps keep the seaweed upright. The leaves of underwater aquatic plants are also softer then above ground plants. This softness allows the plant to move easily with the waves without breaking.

Q 49.B 







Karez system o One of the most unique features of Bidar is the historic ‗Karez‘ system (also known as Qanat) which is a water harnessing technology that originated in Iran/Persia. It was built by Bahmani kings in 15th Century by the Bahmani kings in Bidar, Gulbarg and Bijapur in Karnataka. o These are underground canals built to underground water streams which are meant to provide drinking water to civilian settlements and garrison inside the Bidar fort. This system was necessary in a city like Bidar where the soil was rocky and drilling wells to accommodate drinking water was difficult. Hence, opton (b) is the correct answer. o Recently, Suranga Bawadi in Bijapur Karnataka which is an integral part of the ancient Karez System has entered the World Monuments Watch List under ―Ancient Water System of the Deccan Plateau‖ of the World Monument Funds. Ahar Pynes o It is a traditional floodwater harvesting systems indigenous to South Bihar and have been the most important source of irrigation in this region. Ahars are reservoirs with embankments on three sides and are built at the end of drainage lines such as rivulets or artificial works like pynes. o Pynes are diversion channels led off from the river for irrigation purposes and for impounding water in the ahars. It is mostly to the credit of these that paddy cultivation has been possible in this otherwise relatively low rainfall area, when compared to North Bihar. The system attained its highest development in the district of Gaya. Khadin o Also called a dhora, is an ingenious construction designed to harvest surface runoff water for agriculture. Its main feature is a very long (100-300 m) earthen embankment built across the lower hill slopes lying below gravelly uplands. Sluices and spillways allow excess water to drain off. It is based on the principle of harvesting rainwater on farmland and subsequent use of this water-saturated land for crop production. o First designed by the Paliwal Brahmins of Jaisalmer, western Rajasthan in the 15th century, this system has great similarity with the irrigation methods of the people of Ur (present Iraq) around 4500 BC and later of the Nabateans in the Middle East. A similar system is also reported to have been practised 4,000 years ago in the Negev desert, and in southwestern Colorado 500 years ago. Eri o Belonging to the state of Tamil Nadu, it is one of the oldest water management systems in India. Still widely used in the state, it act as flood-control systems, prevent soil erosion and wastage of runoff during periods of heavy rainfall, and also recharge the groundwater.

Q 50.C Canary Current (Cold Current)  Canary Current, also called Canaries Current, part of a clockwise-setting ocean-current system in the North Atlantic Ocean. 18

www.visionias.in

©Vision IAS



It branches south from the North Atlantic Current and flows southwestward along the northwest coast of Africa as far south as Senegal before turning westward to eventually join the Atlantic North Equatorial Current.  The cool temperature of the water is produced by upwelling caused by offshore winds from the continent. As the current flows around the Canary Islands, it helps to lessen the heating effect of the Sahara to the east. The thermal mixing creates excellent fishing grounds in the region. Falkland current (Cold Current)  The cold waters of the Antarctic Sea flows in the form of Falkland cold current from south to north along the eastern coast of South America up to Argentina.  This current becomes most extensive and developed near 30°S latitude.  This current also brings numerous icebergs from the Antarctic area to the South American coast. Kuroshio Current (Warm Current)  This is also known as Kuru Siwo or Japan Current. Most of it lies in the subtropical high-pressure belt and is under the influence of the westerlies. Since it carries the equatorial waters, it is a warm current. Oyashio Current (Cold Current)  Also known as Oya Siwo, Okhotsk or the Kurile current is a cold subarctic current flowing from the Bering Strait. It flows southwards in a counter-clockwise direction and joins the Kuroshio current off the Japanese island of Hokkaido. Q 51.D 

  

  



The Godavari River is an important river in India and it flows from western to southern India before draining into the Bay of Bengal. The catchment area of the river is regarded as one of the biggest in the country. It rises from a place called Trimbak (Trimbakeshwar) located in the Western Ghats in Nashik district in the state of Maharashtra. Hence statement 1 is correct. It is known as the Dakshin Ganga or Vridha Ganga (old Ganga) because of its age, size and length. It is navigable in the delta region. The river is 1,465 km long and ranks as the second longest river in the country (after the Ganges). The drainage basin of the river is present in seven states of India: Chhattisgarh, Maharashtra, Andhra Pradesh, Telangana, Madhya Pradesh, Karnataka, and Orissa UT of Puducherry. Hence statement 2 is correct. Major Tributaries of the Godavari river are Indravati, Pranhita, Penganga, Purna, Duhna, Manjra, Wardha, Wainganga, Sabari, etc. Polavaram Project is an under construction multi-purpose National project on the Godavari River in the West Godavari District and East Godavari District in Andhra Pradesh. Hence statement 3 is correct. In 2014 the government declared the Polavaram project a National project and the ministry kept the ―Stop Work Order‖(a legal device used in the construction industry to suspend work until a decision or an agreement between the contracted parties has been reached) in abeyance to allow the construction works. It is designed to overcome the deficit in water in the country. The project is entirely funded by the Centre and is also known as National River-Linking Project. In June 2019, Union Environment Ministry had given two years of extension and allowed the construction works related to Polavaram Multipurpose Project. .

Q 52.B 

Volcanic landforms are divided into extrusive and intrusive landforms based on weather magma cools within the crust or above the crust.  Extrusive landforms are formed from material thrown out during volcanic activity. The materials thrown out during volcanic activity includes lava flows, pyroclastic debris, volcanic bombs, ash and dust and gases such as nitrogen compounds, sulphur compounds and minor amounts of chlorine, hydrogen and argon. Examples of Extrusive lanforms are:  Conical Vent and Fissure Vent 19

www.visionias.in

©Vision IAS

      

Mid-Ocean Ridges Composite Type Volcanic Landforms Shield Type Volcanic Landforms Fissure Type Flood Basalt Landforms Caldera Cinder cone A Cinder cone has the features of a steep conical hill with loose pyroclastic fragments which include volcanic clinkers, cinder, volcanic ash (scoria) around the vent.  Cinder cone volcanoes are made entirely of the loose grainy cinders, and they lack lava. Cinder cone usually has very steep sides along with a small crater on its top. They are small volcanoes.  The intrusive igneous rocks or plutonic rocks are formed when the Magma cools within the earth's crust and does not erupt to the surface. Various forms of intrusive igneous rocks are formed due to the intrusive activity of volcanoes. Examples of intrusive landforms are:  Batholiths  Laccoliths  Lapolith  Phacolith  Sills  Dykes  Sills are the intrusive igneous rocks which are formed by the solidified and near horizontal lava layers inside the earth. The thinner deposits of these rocks are called sheets, while the thicker horizontal deposits are known as sills.  When the Magma moves upwards through the cracks and fissures, and solidifies almost perpendicularly to the earth s surface, developing a wall like structure, they are known as dykes. Dykes are the most common intrusive igneous rocks in Western Maharashtra and other parts of Deccan traps. Q 53.C  







20

Option (c) is the correct answer. Mustard: o It is predominantly grown in the northern belt of India, its production has made rapid progress in the last decade. It is a Rabi crop. o The biggest advantage is that they can be grown in a wide range of agro-climatic conditions. In India, rapeseed and mustard are grouped together. Amongst the nine major oilseeds cultivated in India, they come second, only after groundnut. The harvest usually takes place in March or April. o The oil content in rapeseed and mustard is between 36 and 42 %. Indian mustard has a pungent flavour and is often used as a spice in the varied Indian cuisine. Flaxseed: o Flax is a herbaceous annual. o The plant is adaptable to a variety of soils and climates but grows best in well-drained sandy loam and in temperate climates. o In most areas planting of the same land with flax is limited to once in six years to avoid soil exhaustion. o Cool moist growing seasons produce the most-desirable fibre. o Regular flaxseed oil contains between 57% and 71% polyunsaturated fats (alpha-linolenic acid, linoleic acid). Sesame Seed - Sesame is a flowering plant naturalized in tropical regions around the world and is cultivated for its edible seeds. o Sesame varieties have adapted to many soil types but thrive best on well-drained, fertile soils of medium texture and neutral pH. However, these have a low tolerance for soils with high salt and water-logged conditions. o Commercial sesame crops require 90 to 120 frost-free days. Warm conditions above 23 °C favour growth and yields. Soybean o It is a leguminous species native to East Asia, widely grown for its edible bean. It needs about 15 to 32 °C for germination but for rapid growth the crop needs a higher temperature. www.visionias.in ©Vision IAS

o

o o

The crop requires about 60-65 cm annual rainfall drought at flowering or just before flowering results in flower and pod drops, while rains during maturity impair the grain quality of soybean. The best soil type is sandy loam having good organic matter content. Two cropping seasons of soybean Kharif and spring. In case of Kharif season most common time of sowing is the onset of monsoon or last week of June to the first week of Jully while spring sowing is done between 15th of February and 15th of March

Q 54.A 

 



Once a haven for migratory birds, the Kanwar lake in Bihar, Asia‘s largest freshwater oxbow lake, is today a dying wetland ecosystem. Kanwar jheel, as it is locally called, is located 22 km north-west of Begusarai town. It is a residual oxbow lake, formed due to meandering of Gandak river, a tributary of Ganga, in the geological past. Kanwar lake was declared a notified area under the Wildlife (Protection) Act of 1972. To check the poaching of birds, it was declared a protected zone by the Bihar state government in 1986; the government of India declared it a bird sanctuary in 1989. The authorities had notified 15,000 acres (one acre equals 0.4 ha) in the area as a wetland, which makes it six times bigger than Keoladeo National Park in Bharatpur, Rajasthan. It is shrinking fast. The lake covered 6,786 ha in 1984, which reduced to 6,043.825 ha in 2004. By 2012, the lake area had reduced to a mere 2,032 ha. The water level in the lake has reduced, there is heavy siltation and eutrophication (when excess algae and plant growth and their decomposition deprive water of available oxygen, causing the death of other organisms) has set in. Barua Sagar Tal is a large lake situated in Barua Sagar near Jhansi in the Indian state of Uttar Pradesh The Udhwa Lake Bird Sanctuary is the only bird sanctuary in the entire state of Jharkhand. This Sanctuary is famous as a stopping point for a large number of migratory birds that come here in winters from Europe and Siberia. Keoladeo National Park or Keoladeo Ghana National Park is formerly known as the Bharatpur Bird Sanctuary in Bharatpur, Rajasthan, India. It is a famous avifauna sanctuary. Keoladeo Ghana National Park is a man-made and man-managed wetland and one of the national parks of India. It is also a World Heritage Site.

Q 55.A 



21

Bushfires and grassfires are common throughout Australia. Grassfires are fast-moving, passing in five to ten seconds and smouldering for minutes. Bushfires are generally slower-moving but have a higher heat output. This means they pass in two to five minutes, but they can smoulder for days. Bushfires are an intrinsic part of Australia's environment. Many of Australia's native plants are fire-prone and very combustible, while numerous species depend on fire to regenerate. Indigenous Australians have long used fire as a land management tool and it continues to be used to clear land for agricultural purposes and to protect properties from intense, uncontrolled fires. The basic factors which determine whether a bushfire will occur include the presence of fuel, oxygen and an ignition source. The fire intensity and speed at which a bushfire spreads will depend on ambient temperature, fuel load, fuel moisture, wind speed and slope angle. o Fuel load: Fuel load describes the amount of fallen bark, leaf litter and small branches accumulating in the landscape. o Fuel moisture: Dry fuel will burn quickly, but damp or wet fuel may not burn at all. As a consequence, the time since rainfall and the amount of rain received is an important consideration in assessing bushfire danger. o Wind speed: Wind acts to drive a fire by blowing the flames into fresh fuel, bringing it to ignition point and providing a continuous supply of oxygen. Wind also promotes the rapid spread of fire by spotting, which is the ignition of new fires by burning embers lofted into the air by wind. o Ambient temperature: The higher the temperature the more likely it is that a fire will start or continue to burn. This is because the fuel is closer to its ignition point at high temperatures and preheated fuel loads burn faster. o Relative humidity: Dry air promotes a greater intensity fire than moist air. Plants become more flammable at low humidity because they release their moisture more easily. www.visionias.in

©Vision IAS

o o o

Slope angle: Fires pre-heat their fuel source through radiation and convection. As a result, fires accelerate when travelling uphill and decelerate travelling downhill. Ignition Source: Bushfires can originate from both human activity and natural causes with lightning the predominant natural source, accounting for about half of all ignitions in Australia. Hence all the options are correct.

Q 56.C 

Our Solar system consists of eight planets. Out of the eight planets, mercury, venus, earth and mars are called as the inner planets as they lie between the sun and the belt of asteroids the other four planets are called the outer planets. Alternatively, the first four are called Terrestrial Planets, meaning earth-like as they are made up of rock and metals, and have relatively high densities. The rest four are called Jovian or Gas Giant planets. Jovian means Jupiter-like. Most of them are much larger than the terrestrial planets and have a thick atmosphere, mostly of helium and hydrogen. Reason for the same are: o The terrestrial planets were formed in the close vicinity of the parent star where it was too warm for gases to condense to solid particles. Jovian planets were formed at quite a distant location. o The solar wind was most intense nearer the sun; so, it blew off lots of gas and dust from the terrestrial planets. The solar winds were not all that intense to cause similar removal of gases from the Jovian planets. Hence statement 1 is correct. o The terrestrial planets are smaller and their lower gravity could not hold the escaping gases. Hence, statement 2 is correct.

Q 57.A   



 

The year 2019 has been an important year for butterfly researchers in India. In January 2019, after a span of 120 long years, researchers at the Sikkim University in Gangtok have also rediscovered the Small Woodbrown butterfly species from Khanchendzonga National Park. Tamil Nadu has a total of 32 species of butterflies endemic to the state. With the latest policy initiative, it has become the fifth Indian state after Maharashtra to bestow state emblem status to one of its colourful insects. Important Butterflies found in India: o Maharashtra (Blue Mormon which is endemic to India (South India) and Sri Lanka), o Uttarakhand (Common peacock which is native to Asia and Australia and is also called Chinese Peacock Butterfly), o Karnataka (Southern birdwing, which is endemic to South India and is India's largest butterfly) and o Kerala (Malabar banded peacock) to bestow a state emblem status to one of its colourful insects. The state has as many as 32 butterfly hotspots – designated as a place where at least 25% of all the available species found in the state – while experts are on track to identify more such zones. Hence only option 1 is correct.

Q 58.D 



22

In large flood and delta plains, rivers rarely flow in straight courses. Loop-like channel patterns called meanders develop over flood and delta plains. Characteristics required for their formation o propensity of water flowing over very gentle gradients to work laterally on the banks. Hence option 1 is correct. o unconsolidated nature of alluvial deposits making up the banks with many irregularities which can be used by water exerting pressure laterally. Hence option 2 is correct. o Coriolis force acting on the fluid water deflecting it like it deflects the wind. Hence option 3 is correct. o When the gradient of the channel becomes extremely low, water flows leisurely and starts working laterally. Slight irregularities along the banks slowly get transformed into a small o curvature in the banks; the curvature deepens due to deposition on the inside of the curve and erosion along the bank on the outside. If there is no deposition and no erosion or undercutting, the tendency to meander is reduced. As meanders grow into deep loops, the same may get cut-off due to erosion at the inflection points and are left as ox-bow lakes. www.visionias.in

©Vision IAS

Q 59.A 





Earth‘s tilted axis causes the seasons. Throughout the year, different parts of Earth receive the Sun‘s most direct rays. So, when the North Pole tilts toward the Sun, it‘s summer in the Northern Hemisphere. And when the South Pole tilts toward the Sun, it‘s winter in the Northern Hemisphere. Hence statement 1 is correct. The Earth‘s revolution not only affects but actually causes the temperature conditions that give us spring, summer, fall and winter seasons. Which season it depends on whether you live in the Northern or Southern Hemisphere because the Earth‘s axis tilts toward one of the two as it moves around the sun. The seasons are always opposite in each hemisphere. Hence statement 2 is correct. Earth's axis is an imaginary pole going right through the centre of Earth from "top" to "bottom." Earth spins around this pole, making one complete turn each day. That is why we have day and night, and Earth's rotation has no effect on seasons. Hence statement 3 is not correct.

Q 60.B 







Peace Pagodas, known as Shanti Stupas, are Buddhist monuments built to inspire peace. A place for individuals of every race and background, the stupas were ideated under the guidance of Japanese monk Nichidatsu Fujii who was heavily influenced by Mahatma Gandhi's commitment to non-violence. Hence statement 1 is not correct. Built around the world especially in war-devastated cities like Hiroshima, Munich and Nagasaki, the pagodas symbolise unity and togetherness. A lot of these great structures were built by locals with the help of other monks and nuns from the Nipponzan-Myohoji Buddhist Order, founded by Fujii. One of the major Shanti Stupa located at Rajgir, Bihar. In series of establishing peace pagodas world over Nichidatsu Fujii established one such peace pagoda at Rajgir which is called Vishwa Shanti Stupa. It is made of spotless white marble and the tallest peace Pagoda in the world standing tall at 400 meters. It houses four statues of Lord Buddha on the four side-corners of the stupa. Hence statement 2 is correct. Recently, the Vishwa Shanti Stupa (World Peace Pagoda) at Rajgir in Bihar celebrated its 50th anniversary which was attended by the President of India.

Q 61.A 







The vulnerability of species is likely a combination of two factors—their endemism and rarity. In the case of lions, wolves, and wild dogs, population size alone is a poor predictor of their local extinction, even when the animals live in protected areas. Generally, the larger the body size of an animal, the longer it lives and the fewer offspring it produces each year. Relatively large animals also tend to have relatively low population densities; thus, a viable population of, say, elephants occupies considerably more space than an equal-sized population of rabbits. Some species have aggregation behaviours that make them vulnerable to disturbance or hunting. For example, bats may congregate in large numbers in particular caves to have their young, making significant portions of their total population especially susceptible when their habitat is disturbed by human visitation or damaged by the cave‘s commercialization or flooding. Species that have low dispersal rates are at a disadvantage because it is unlikely that one population can save another. Because small populations are so much more at risk than large ones, individuals of species that can readily disperse can rescue local populations on the verge of extinction.

Q 62.C 



23

The ENCORE is proposed as an eight-year Multiphase Programmatic Approach (MPA) operation with two overlapping phases of five years each that are anchored in the umbrella India‘s National Coastal Mission. The ENCORE project aims to strengthen integrated coastal zone management in all the coastal states and the Union Territories of India through the following components. o Component 1: Improved Capacity for Decentralized Coastal Management. o Component 2: Improved Protection and Pollution Abatement in Coastal Areas. o Component 3: Project Management, Monitoring, and Evaluation The Program seeks to assist the Government of India (GoI) in enhancing coastal resource efficiency and resilience, by building collective capacity (including communities and decentralized governance)for adopting and implementing Integrated Coastal Zone Management (ICZM) approaches. www.visionias.in

©Vision IAS





Since ENCORE Program is funded under the Investment Project Financing (IPF) instrument of the World Bank, its Operational Policies (OPs) and Bank Procedures (BPs) are applicable for the project. Hence both the statements are correct. Join Telegram Channel For Clean UPSC Materials

Share and Subscribe our telegram channel Q 63.B https://t.me/UPSC_PDF or @UPSC_PDF Outwash plain  An outwash plain, also called a sander, is a plain formed of glacial sediments deposited by meltwater outwash at the terminus of a glacier.  As it flows, the glacier grinds the underlying rock surface and carries the debris along. The meltwater at the snout of the glacier deposits its load of sediment over the outwash plain, with larger boulders being deposited near the terminal moraine, and smaller particles travelling further before being deposited.  Sanders is common in Iceland where geothermal activity accelerates the melting of ice flows and the deposition of sediment by meltwater. Cirques  Cirques are the most common of landforms in the glaciated mountains. The cirques quite often are found at the heads of glacial valleys. The accumulated ice cuts these cirques while moving down the mountain tops.  They are deep, long and wide troughs or basins with very steep concave to vertically dropping high walls at its head as well as sides. A lake of water can be seen quite often within the cirques after the glacier disappears. Such lakes are called cirque or tarn lakes. There can be two or more cirques one leading into another down below in a stepped sequence. Playas  Plains are by far the most prominent landforms in the deserts. In basins with mountains and hills around and along, the drainage is towards the centre of the basin and due to gradual deposition of sediment from basin margins, a nearly level plain forms at the centre of the basin.  In times of sufficient water, this plain is covered up by a shallow water body. Such types of shallow lakes are called as playas where water is retained only for short duration due to evaporation and quite often the playas contain good deposition of salts. The playa plain covered up by salts is called alkali flats.

Q 64.B 

24

DTrack, as flagged by cyber-security firm Kaspersky, is used by hackers to attack financial and research centres in India. It's earlier version ATMDtrack was designed to hack ATMs in India. Nuclear Power www.visionias.in

©Vision IAS



Corporation of India Limited (NPCIL) has admitted that one of its computers has been attacked by malware. The malware was designed to be planted on the victim‘s ATMs, where it could read and store the data of cards that were inserted into the machines. Later a version of it was used to attack the banking system in South Korea as well as for the infamous WannaCry ransom worm attacks across the globe.

Q 65.C 













25

Statement 1 is not correct: The troposphere is the lowermost layer of the atmosphere. Its average height is 13 km and extends roughly to a height of 8 km near the poles and about 18 km at the equator. This layer contains dust particles and water vapour. All changes in climate and weather take place in this layer. The temperature in this layer decreases at the rate of 1° C for every 165m of height. The zone separating the troposphere from the stratosphere is known as the tropopause. The air temperature at the tropopause is about minus 800C over the equator and about minus 45 degrees C over the poles. The temperature here is nearly constant. The stratosphere is found above the tropopause and extends up to a height of 50 km. One important feature of the stratosphere is that it contains the ozone layer. This layer absorbs ultra-violet radiation and shields life on the earth from the intense, harmful form of energy. In this layer, the temperature starts increasing with altitude. The mesosphere lies above the stratosphere, which extends up to a height of 80 km. In this layer, once again, the temperature starts decreasing with the increase in altitude and reaches up to minus 100° C at the height of 80 km. The upper limit of the mesosphere is known as the mesopause. The ionosphere is located between 80 and 400 km above the mesopause. It contains electrically charged particles known as ions, and hence, it is known as the ionosphere. Radio waves transmitted from the earth are reflected back to the earth by this layer. Again in this layer, the temperature starts increasing with height. Statement 2 is not correct: In the stratosphere, the temperatures drop with increasing altitude. Because of this temperature stratification, there is little convection and mixing in the stratosphere, so the layers of air there are quite stable. Commercial jet aircraft fly in the lower stratosphere to avoid the turbulence which is common in the troposphere below. Statement 3 is correct: The ionosphere is located between 80 and 400 km above the mesopause. It contains electrically charged particles known as ions, and hence, it is known as the ionosphere. Radio waves transmitted from the earth are reflected back to the earth by this layer. Again in this layer, the temperature starts increasing with height.

www.visionias.in

©Vision IAS

Q 66.A 



The South Pacific islands of Bougainville have overwhelmingly voted to be independent from Papua New Guinea in a historic poll that has emboldened separatists as they start negotiating the terms of their sovereignty This referendum was held between 23 November and 7 December 2019, with results being declared on 11 December. The referendum question was a choice between greater autonomy within Papua New Guinea, or full independence. Of the valid votes, 98.31% were in favour of full Independence. The vote is not binding; the Government of Papua New Guinea has the final say on the status of Bougainville.

Q 67.B  





The tidal range is the height difference between high tide and low tide. Once in a month, when the moon‘s orbit is closest to the earth (perigee), unusually high and low tides occur. During this time the tidal range is greater than normal. Two weeks later, when the moon is farthest from earth (apogee), the moon‘s gravitational force is limited and the tidal ranges are less than their average heights. When the earth is closest to the sun (perihelion), around 3rd January each year, tidal ranges are also much greater, with unusually high and unusually low tides. When the earth is farthest from the sun (aphelion), around 4th July each year, tidal ranges are much less than average. Hence perigee and perihelion are associated with above-normal tidal ranges.

Q 68.A 





  

Bandhavgarh National Park is one of the national parks of India, located in the Umaria district of Madhya Pradesh. Bandhavgarh, with an area of 105 km2, was declared a national park in 1968 and then became Tiger Reserve in 1993. This park has a large biodiversity. The density of the tiger population at Bandhavgarh (8 Tiger per square km)is one of the highest known in India. The park has a large breeding population of leopards and various species of deer In November 2019, elephants in herds, 38 of them, wandered into the forests of Bandhavgarh looking for food and water, like each year. A year on, they have stayed back in Madhya Pradesh for the first time over seasons and even bred two new calves, choosing not to return to the withering forests of north Chhattisgarh. The All India Tiger Estimation Report 2018 said the country has emerged as of one of the biggest and safest habitats for tigers in the world. It revealed that Madhya Pradesh was the country's "tiger state" with 526 of the big cats. Bandipur National Park, a forested reserve in the southern Indian state of Karnataka, is known for its small population of tigers. Jim Corbett National Park is a forested wildlife sanctuary in northern India‘s Uttarakhand State. Rich in flora and fauna, it‘s known for its Bengal tigers. Nagarhole National Park is a wildlife reserve in the South Indian state of Karnataka. It is a part of the Nilgiri Biosphere Reserve.

Q 69.C 

26

Statements 1 and 3 are not correct: The European Green Deal was published towards the end of Conference of the Parties (COP) 25, the international climate change conference which was held in Madrid earlier this month. This deal is NOT a part of the COP25 documents. While the COP 25 meeting was still on, the European Union, whose 28 member countries are together the third-largest emitter of greenhouse gases in the world after China and the United States, came up with an announcement on additional measures it would on climate change called the European Green Deal, the EU announcement was hailed as a major step forward, even though it needs complementary efforts from other countries to make a significant impact. India is NOT a party to this deal as this is confined to the initiatives of the European countries.

www.visionias.in

©Vision IAS





Statement 2 is correct: The overarching objective of the green deal is to provide a roadmap for the transformation of the European economy and to ensure that Europe is the first continent to be carbon neutral by 2050. There are many other specific proposals in the green deal but some of the most notable are: o cutting carbon emissions to zero by 2050, and by 50% (if not 55%) compared to 1990 levels. o a new chemicals strategy is expected in 2020, which will have a focus on preventing air and water pollution o a strategy will likely be launched in 2020 called 'farm to fork' which will improve the sustainability of the production and distribution of food

Q 70.B 







Environment Education Awareness and Training (EEAT) is an established central sector scheme of the Ministry of Environment, Forest and Climate Change continuing since 1983-84 and aims to promote environmental awareness and mobilize student‘s participation for environment conservation. Under the scheme, National Green Corps (NGC) ‗Ecoclub‘ programme was initiated in 2001-2002 with the objective to impart knowledge to school children through hands-on experience, about their immediate environment, interactions within it and the problems therein. The programme aims to inculcate proper attitude towards the environment and sensitize children on issues related to environment and development. Recently the first Annual Meet of the State Nodal Agencies implementing the National Green Corps ‗Ecoclub‘ programme of the Ministry of the Environment, Forest and Climate Change (MoEF&CC), was organized by the Ministry‘s Environment Education Division. Also, the National Green Corps scheme, which was closed for over a decade, has been revived for Jammu and Kashmir. J&K PCB put efforts for the revival of the scheme in the State Nodal Agency (SNA) meet organized by MoEFCC at Kevadia in Gujarat. Accordingly, the Environment and Forests Ministry has approved the release of Grant-in-Aid of 2.60 crore rupees to J&K PCB. Under this scheme, the PCB would provide financial assistance to 3,742 Eco Clubs of registered schools in both the regions of the Union Territory along with 96 Eco Clubs of registered colleges, for which funds would soon be released via Direct Bank Transfer by the Board shortly.

Q 71.D  









27

An urban heat island (UHI) is an urban area or metropolitan area that is significantly warmer than its surrounding rural areas due to human activities. Around the world, cities are often a few degrees hotter than surrounding rural areas. A recent study verified that across the world, the intensity of this so-called urban heat island effect depends on a city‘s population and how much rainfall it receives annually. One key factor, according to lead researcher Gabriele Manoli, is how much water is exchanged with the atmosphere by a city compared with the surrounding land. ―Evapotranspiration is the process of transferring water from the soil into the atmosphere, regulated by vegetation,‖ Manoli, an environmental engineer at University College London, told Eos. ―This process has a cooling effect on the Earth‘s surface.‖ Adding green space to a city is a common strategy to mitigate urban heat. If a city has more greenery than the surrounding area, the urban heat island (UHI) intensity will be relatively mild, Manoli‘s team found. But the study suggests that if a city sits in a lush and verdant region, ―vegetation still helps, but you need many more trees to get the same effect,‖ he said. Regionally, ―it was known that there are two general trends: The intensity of urban heat islands increases with population and…with increasing precipitation in the region,‖ Manoli explained. ―Population is a proxy for the size of a city, its form, activities, and infrastructures, while precipitation is a proxy for the local climate and vegetation characteristics.‖ According to some researches, heat island intensity increases with the city-size and compact nature of the city. www.visionias.in

©Vision IAS

Q 72.B 

 



All of above hills are part of the Purvanchal hills or Eastern hils. Their direction from South-West to North-East direction is given below: o Dafla hills o Miri hills o Abor hills o Mishmi hills Daffla Hills: It is situated to the north of the Tezpur and North Lakhimpur, and is bounded on the west by the Aka Hills and on the east by the Abor Range Abor Hills: It is located in a region of Arunachal Pradesh in the far north-east of India, near the border with China. It is bordered by the Mishmi Hills and Miri Hills. This region is drained by the Dibang River, a tributary of the Brahmaputra. Mishmi Hills: These hills are located in the southward extension of the Great Himalayan ranges and its northern and eastern parts touches China.

Q 73.D 

28

The Western Ghats are the mountainous faulted and eroded edge of the Deccan Plateau. Geologic evidence indicates that they were formed during the break-up of the supercontinent of Gondwana some 150 million years ago. Hence statement 1 is correct. www.visionias.in

©Vision IAS

  

   

The Western Ghat mountain range is India‘s second-largest mountain range after the Himalayas. Its length is about 1500 Km. This range extends from Tapi river valley to the Nilgiri hills. It is parallel to the western coast of the Indian peninsula, traversing the states of Kerala, Tamil Nadu, Karnataka, Goa, Maharashtra and Gujarat. The Western Ghats are in the form of block mountains. In the process of separation of India from Africa, a fault valley has been formed in the form of Arabian Sea, and the Western Ghats have been left as the escarpment of this fault valley. The steep slope of the Western Ghats of peninsular India shows that fault. Anamudi (2,695 m), the highest peak of Peninsular plateau is located on the Anaimalai Hills of the Western Ghats followed by Dodabetta (2,637 m) on the Nilgiri hills. Hence statement 2 is correct. Most of the Peninsular rivers have their origin in the Western Ghats. Deccan Plateau is bordered by the Western Ghats in the west, Eastern Ghats in the east and the Satpura, Maikal range and Mahadeo hills in the north. Western Ghats are locally known by different names such as Sahyadri in Maharashtra, Nilgiri hills in Karnataka and Tamil Nadu and Anaimalai hills and Cardamom hills in Kerala. Hence statement 3 is correct.

Q 74.C 



Q 75.B 

The government has recently launched the National Broadband Mission to fast track growth of digital communications infrastructure, bridge the digital divide, facilitate digital empowerment and inclusion and provide affordable and universal access of broadband for all. Some of the objectives of the Mission include -: o Broadband access to all villages by 2022. Hence, statement 1 is correct. o Facilitate universal and equitable access to broadband services for across the country and especially in rural and remote areas o Laying of incremental 30 lakhs route km of Optical Fiber Cable and increase in tower density from 0.42 to 1.0 tower per thousand of population by 2024 o Significantly improve quality of services for mobile and internet o Develop innovative implementation models for Right of Way (RoW) and to work with States/UTs for having consistent policies pertaining to expansion of digital infrastructure including for RoW approvals required for laying of OFC o Develop a Broadband Readiness Index (BRI) to measure the availability of digital communications infrastructure and conducive policy ecosystem within a State/UT. The National Digital Communications Policy – 2018 (NDCP-18) recognizes digital communications infrastructure and services as key enablers and critical determinants of India's growth and well-being.One of the objectives of NDCP-18 is to provision 'Broadband for All' so that the resulting benefits of widespread, equitable and inclusive growth are enjoyed by all. In order to operationalize 'Broadband for All' the ―National Broadband Mission‖ has been launched to secure universal broadband access. Hence, Join Telegram Channel For Clean UPSC Materials statement 2 is correct.

Share and Subscribe our telegram channel https://t.me/UPSC_PDF or @UPSC_PDF Santhali is the most widely-spoken language of the Munda, a subfamily of the Austroasiatic languages. It is mainly spoken in the Indian states of Assam, Bihar, Jharkhand, Mizoram, Odisha, Tripura and West Bengal. In addition, it is also spoken in Bangladesh and Nepal. It was added to the eighth schedule of the Indian Constitution by the ninety-second constitutional amendment act. The script for the Santhali language is referred to as Ol-Chiki which was developed by Pandit Raghunath Murmu. Recently, Santhali language made its debut in Rajya Sabha when a lawmaker from Odisha used it to raise a matter of urgent public importance. Hence option (b) is the correct answer.

Q 76.A 

29

The distribution of atmospheric pressure is not uniform over the earth‘s surface. It varies vertically as well as horizontally. www.visionias.in

©Vision IAS













 



The distribution of atmospheric pressure across the latitudes is termed as global horizontal distribution. This distribution is characterised by presence of distinctly identifiable zones of homogeneous pressure regimes or ‗pressure belts‘. On the earth‘s surface, there are in all seven pressure belts. The seven pressure belts are: equatorial low, the sub-tropical highs, the sub-polar lows, and the polar highs. Except the equatorial low, all others form matching pairs in the northern and southern hemispheres. On the basis of mode of genesis pressure belts are divided into two broad category: o thermally induced pressure belt (equatorial low pressure belt and polar high pressure belt). Hence statement 1 is not correct. o dynamically induced pressure belt (eg. Subtropical high pressure belt and subpolar low pressure belt) Equatorial Low Pressure Belt or ‗Doldrums‘: o This belt lies between 5°N and 5°S latitudes, although the width may vary between 5°N and 5°S and 20°N and 20°S. Due to intense heating, air gets warmed up and rises over the equatorial region and produces the equatorial low pressure belt. This belt is characterised by extremely low pressure with calm conditions. o Surface winds are generally absent since winds approaching this belt begin to rise near its margin. Thus, only vertical currents are found. This belt happens to be the zone of convergence of trade winds from two hemispheres from sub-tropical high pressure belts. This belt is also called the Doldrums, because of the extremely calm air movements. Hence statement 2 is correct. Sub-Tropical High Pressure Belt or ‗Horse Latitudes‘: o It lies between 25°-30° North and South in northern hemisphere and southern hemisphere . Hence statement 3 is not correct. o The high pressure along this belt is due to subsidence of air coming from the equatorial region which descends after becoming heavy. o The high pressure is also due to the blocking effect of air at upper levels because of the Coriolis force. The subsiding air is warm and dry, therefore, most of the deserts are present along this belt, in both hemispheres. o The descending air currents feed the winds blowing towards adjoining low pressure belts. A calm condition with variable and feeble winds is created in these high pressure belts, called horse latitudes. In early days, the sailing vessels with a cargo of horses found it difficult to sail under such calm conditions. They used to throw horses into the sea when fodder ran out. This belt is frequently invaded by tropical and extra-tropical disturbances. Sub-Polar Low Pressure Belt: This belt is located between 45°N and S latitudes and the Arctic and the Antarctic circles. The low pressure exists along this belt due to ascent of air as a result of convergence of westerlies and polar easterlies. During winter, because of a high contrast between land and sea, this belt is broken into two distinct low centres—one in the vicinity of the Aleutian Islands and the other between Iceland and Greenland. During summer, a lesser contrast results in a more developed and regular belt. Also, due to a great contrast between the temperatures of the winds from sub-tropical and polar source regions, cyclonic storms or ‗lows‘ are produced in the region. Polar High: The lowest temperatures are found over the poles, which cause subsidence of air and hence the polar highs. The polar highs are small in area and extend around the poles.

Q 77.D  

30

Krishna-Godavari basin, located in eastern India, is considered to hold the largest shale gas reserves in the country. The Cambay basin in Gujarat is the largest basin in the country.

www.visionias.in

©Vision IAS

Q 78.D 

Groundwater depletion is primarily caused by sustained groundwater pumping. Some of the negative effects of groundwater depletion are : o Lowering of the Water Table: Excessive pumping can lower the groundwater table, and cause wells to no longer be able to reach groundwater. o Increased Costs: As the water table lowers, the water must be pumped farther to reach the surface, using more energy. In extreme cases, using such a well can be cost-prohibitive. o Reduced Surface Water Supplies: Groundwater and surface water are connected. When groundwater is overused, the lakes, streams, and rivers connected to groundwater can also have their supply diminished. Hence statement 1 is correct. o Land Subsidence: Land subsidence occurs when there is a loss of support below the ground. This is most often caused by human activities, mainly from the overuse of groundwater, when the soil collapses, compacts, and drops. Hence statement 2 is correct. o Water Quality Concerns: Excessive pumping in coastal areas can cause saltwater to move inland and upward, resulting in saltwater contamination of the water supply. In fact, over withdrawals in some states like Rajasthan, and Maharashtra has increased fluoride concentration in groundwater, and this practice has led to increase in the concentration of arsenic in parts of West Bengal and Bihar. Hence statement 3 is correct.

Q 79.A 

 

Biomagnification, also known as bioamplification or biological magnification, is any concentration of a toxin, such as pesticides, in the tissues of tolerant organisms at successively higher levels in a food chain. Biomagnification is a cumulative increase in the concentrations of a persistent substance (e.g. pesticides, metals, etc.) as it moves up the food chain. This occurs when agricultural, industrial, or human waste pours into the ocean directly or via rivers, sewage, etc. This increase can occur as a result of Persistence – where the substance cannot be broken down by environmental processes and waterinsolubility. Hence statement 1 is correct. Bioaccumulants are substances that increase in concentration in living organisms as they take in contaminated air, water, or food because the substances are very slowly metabolized or excreted. In order to biomagnification to occur, the pollutant must be long-lived, mobile and soluble in fats. If pollutant is short-lived, it will be broken down before it can become dangerous. Hence statement 2 is not correct.

Q 80.C 

31

Statement 1 is correct: Masses of ice moving as sheets over the land (continental glacier or piedmont glacier if a vast sheet of ice is spread over the plains at the foot of mountains) or as linear flows down the slopes of mountains in broad trough-like valleys (mountain and valley glaciers) are called glaciers. www.visionias.in

©Vision IAS



Statement 2 is correct: The movement of glaciers is slow unlike water flow. The movement could be a few centimetres to a few metres a day or even less or more. Glaciers move basically because of the force of gravity and pressure from the weight of overlying ice. Because of friction caused by sheer weight of the ice, erosion by glaciers is tremendous and Glaciers can cause significant damage to even unweathered rocks and can reduce high mountains into low hills and plains.

Q 81.D 



An invasive plant alters the habitat. It draws in nutrients and moisture, outcompetes native species and paves the way for further invasions by microorganisms, pests and pathogens. Proliferation of these IAS threatens native biodiversity, subverts natural plant succession, changes structure and composition of communities, and impairs ecosystem services severely. Some of the floral species posing concern in India as Invasive Alien Species are: o Prospis julflora: Vilayti Kikar (Prosopis juliflora) It causes degradation to the native flora and depletes soil nutrition. It grows well in arid conditions and has the ability to tolerate salinity. Thick canopy of the tree suppresses growth and regeneration of other tree species lowering the species richness of the area. Its thorny branches make it difficult for foraging by livestock. It has a strong seed dispersal mechanism and impacts native plant growth by preventing sunlight and releasing chemicals. o Parthenium hysterophorus- colloquially known as Congress grass, is believed to have been introduced into India and Australia from North America. It has emerged as the seventh most devastating weed in Africa, Asia, and Australia responsible for severe human and animalhealth issues, such as dermatitis, asthma and bronchitis, agricultural losses, besides being a big problem for biodiversity. o Lantana camara introduced in India, its diverse and broad geographic distribution is a reflection of its wide ecological tolerance. It occurs in diverse habitats and on a variety of soil types. Lantana grows as impenetrable thickets that can suppress the growth of native species. Lantana threatens natural habitats and native flora and fauna (FAO). o Water hyacinth (Eichhornia crassipes) with its massive proliferation minimises availability of open water surface. Large scale invasion prevents entry of sunlight and significantly alters airwater interface and adversely affects the productivity of the ecosystem.

Q 82.A 



 

Red soil develops on crystalline igneous rocks in areas of low rainfall in the eastern and southern part of the Deccan Plateau. It is the Laterite Soil which develops in areas with high temperatures and high rainfall. Hence, statement 1 is not correct. Alluvial soils are widespread in the northern plains and the river valleys. These soils cover about 40 percent of the total area of the country. Hence, Alluvial soils form the largest group of soils in India and not the Red and Yellow soils. Hence, statement 2 is not correct. The soil develops a reddish color due to a wide diffusion of iron in rocks. It looks yellow when it occurs in a hydrated form. Hence, statement 3 is correct. The fine-grained red and yellow soils are normally fertile, whereas coarse-grained soils found in dry upland areas are poor in fertility. They are generally poor in nitrogen, phosphorous and humus.

Q 83.D 





32

Considering the commercial exploitation of the turtles and tortoises as well as the extent of involvement of illegal wildlife traders in the illegal trade of live specimens Wildlife Crime Control Bureau (WCCB) envisaged a species specific operation code named ―OPERATION SAVE KURMA‖ to focus specifically on the major states involved in the poaching, transportation and illegal trade of live turtles and tortoises. Hence pair 1 is not correctly matched. An operation to drag the attention of the enforcement agencies within the country to focus their attention towards the illegal wildlife trade in Tibetan antelope (Shahtoosh) species which are generally neglected while doing enforcement activities- ―Operation Soft Gold‖ was envisaged and conducted by WCCB from 1st Oct 2018 to 31st Mar 2019. Operation Thunderbird is the code-name for INTERPOL‘s multi-national and multi-species enforcement operation. In order to carry forward the successes of Operation Thunderbird, INTERPOL www.visionias.in ©Vision IAS



 



coordinated enforcement operation code-named ―Operation THUNDERSTORM‖ was launched in which WCCB coordinated the countrywide multi-species wildlife‖ operation during the period. Hence pair 2 is not correctly matched. To encounter the menace of the illegal trade through e-commerce platform, WCCB organized and coordinated an enforcement operation named ―Operation WILDNET‖ to drag the attention of the enforcement agencies within the country to focus their attention on the ever-increasing illegal wildlife trade over the internet. Hence pair 3 is not correctly matched. Operation ―BIRBIL‖ aimed to collect intelligence pertaining to the illegal trade in all Birds and cat species. The Bureau coordinated a wildlife enforcement operation code-named ―Operation LESKNOW‖. The Operation was intended to drag attention of the enforcement agencies within the country to focus their attention towards the illegal wildlife trade in lesser-known species. The Bureau envisaged and coordinated another wildlife enforcement operation code named ―Operation LESKNOW-II‖. The Operation was intended to drag attention of the enforcement agencies within the country towards the illegal wildlife trade in lesser known species such as Deer, Wild Boar, Jackal, Mongoose, Monitor Lizard, Sea cucumber and Pangolin etc.

Q 84.B 



 

The Chambal rises near Mhow in the Malwa plateau of Madhya Pradesh and flows northwards through a gorge up wards of Kota in Rajasthan. The Chambal and its tributaries sap the Malwa area of northwestern Madhya Pradesh, and at the same time its tributary, the Banas, which originates in the Aravalli Range, saps the southeast part of Rajasthan. The Chambal is famous for its badland topography called the Chambal ravines. There are a number of Dams on the Chambal River. The river waters are used for hydroelectric power creation. These dams are the Gandhi Sagar Dam, Jawahar Sagar Dam, and the Rana Pratap Sagar dam. Koteshwar Dam is a gravity dam on the Bhagirathi River, located downstream of the Tehri Dam in Tehri District, Uttarakhand, India. Bheemgarh dam or Bhimgarh, officially the Upper Wainganga Dam, is built across the Wainganga river in the Seoni district of the Indian state of Madhya Pradesh.

Q 85.C 







These reasons for migration can be put into two broad categories : (i) push factors, these cause people to leave their place of residence or origin; and (ii) pull factors, which attract the people from different places. In India people migrate from rural to urban areas mainly due to poverty, high population pressure on the land, lack of basic infrastructural facilities like health care, education, etc. Apart from these factors, natural disasters such as, flood, drought, cyclonic storms, earthquake, tsunami, wars and local conflicts also give extra push to migrate. On the other hand, there are pull factors which attract people from rural areas to cities. The most important pull factor for majority of the rural migrants to urban areas is the better opportunities, availability of regular work and relatively higher wages. Better opportunities for education, better health facilities and sources of entertainment, etc. are also quite important pull factors. Hence option (c) is the correct answer.

Q 86.A 

The Ministry of Environment, Forest and Climate Change notified the E-Waste Management Rules, 2016 on 23 March 2016 in supersession of the e-waste (Management & Handling) Rules, 2011. E-Waste (Management) Rules, 2016  Manufacturer, dealer, refurbisher and Producer Responsibility Organization (PRO) have been introduced as additional stakeholders in the rules. The applicability of the rules has been extended to components, consumables, spares and parts of EEE in addition to equipment as listed in Schedule I.  Compact Fluorescent Lamp (CFL) and other mercury containing lamp brought under the purview of rules. 33

www.visionias.in

©Vision IAS

 



 



Collection mechanism based approach has been adopted to include collection centre, collection point, take back system etc for collection of e - waste by Producers under Extended Producer Responsibility (EPR). Deposit Refund Scheme has been introduced as an additional economic instrument wherein the producer charges an additional amount as a deposit at the time of sale of the electrical and electronic equipment and returns it to the consumer along with interest when the end - of - life electrical and electronic equipment is returned. The e-waste exchange as an option has been provided in the rules as an independent market instrument offering assistance or independent electronic systems offering services for sale and purchase of e-waste generated from end-of-life electrical and electronic equipment between agencies or organizations authorised under these rules. The roles of the State Government has been also introduced in the Rules in order to ensure safety, health and skill development of the workers involved in the dismantling and recycling operations. Liability for damages caused to the environment or third party due to improper management of e-waste including provision for levying financial penalty for violation of provisions of the Rules has also been introduced. Urban Local Bodies (Municipal Committee/Council/Corporation) has been assign the duty to collect and channelized the orphan products to authorized dismantler or recycler.

Q 87.D 

The northern plains are formed by the alluvial deposits brought by the rivers – the Indus, the Ganga and the Brahmaputra. These plains extend approximately 3,200 km from the east to the west. The average width of these plains varies between 150-300 km. The maximum depth of alluvium deposits varies between 1,000-2,000 m. From the north to the south, these can be divided into three major zones: the Bhabar, the Terai and the alluvial plains. The alluvial plains can be further divided into the Khadar and the Bhangar. Hence option (d) is correct. o Bhabar is a narrow belt ranging between 8-10 km parallel to the Shiwalik foothills at the break-up of the slope. As a result of this, the streams and rivers coming from the mountains deposit heavy materials of rocks and boulders, and at times, disappear in this zone. o South of the Bhabar is the Terai belt, with an approximate width of 10-20 km where most of the streams and rivers re-emerge without having any properly demarcated channel, thereby, creating marshy and swampy conditions known as the Tarai. This has a luxurious growth of natural vegetation and houses a varied wildlife. o The south of Terai is a belt consisting of old and new alluvial deposits known as the Bhangar and Khadar respectively. These plains have characteristic features of mature stage of fluvial erosional and depositional landforms such as sand bars, meanders, oxbow lakes and braided channels. The Brahmaputra plains are known for their riverine islands and sand bars. Most of these areas are subjected to periodic floods and shifting river courses forming braided streams.

Q 88.A 



34

The bar-headed goose (Anser indicus) is a goose that breeds in Central Asia in colonies of thousands near mountain lakes and winters in South Asia, as far south as peninsular India. It lays three to eight eggs at a time in a ground nest. The bar-headed goose (Anser indicus) flies over the tallest peaks of the Himalayas as it migrates from India to Mongolia each year. When oxygen levels in the thin air dip as low as 7 percent, the bird‘s metabolism likewise dips to accommodate, yet its wings beat just as fast as before. Bar-headed geese www.visionias.in

©Vision IAS





migrate above 26,000 feet. A better understanding of their efficient use of oxygen could have implications for human medical treatment. Scientists have long known that bar-headed geese boast an enhanced ability to bind oxygen to their haemoglobin. Studies have demonstrated that bar-headed geese have more capillaries around individual cells in their pectoral muscles than barnacle geese and other related species that don‘t fly at such high altitudes. Those cells are also dense with mitochondria, which use oxygen to supply energy to the cell. Additionally, bar-headed geese hug the terrain as they fly over the Himalayas, taking advantage of the relatively more oxygen-rich air over valleys. They are popularly known as Astronauts of the bird world.

Q 89.A  







Some of the landmark environmental legislation in India include: The Government of India has taken a pioneering initiative for conserving its national animal, the Tiger, by launching the ‗Project Tiger‘ in 1973. From Nine Tiger Reserves since its formative years, the Project Tiger coverage has increased to fifty at present, spread out in 18 tiger range states. This amounts to around 2.21% of the geographical area of our country. Project Tiger was launched in 1973 on the basis of the recommendations of a special task force of the Indian Board of Wildlife to (i) ensure maintenance of available population of tiger in India, and (ii) preserve the areas of such biological importance as a national heritage for the benefit, education and enjoyment of the people. Recognizing the value and need for conservation of the wildlife wealth of the State, Tamil Nadu Government enacted separate law for protection of our valuable animals (The Tamil Nadu Wild Elephants Preservation Act 1873 and The Wild Birds and Animals (Protection) Act,1912), much before enactment of the Wildlife (Protection) Act, 1972. Vedanthangal Bird Sanctuary was declared way back in 1936, which was the ―first‖ Birds Sanctuary in India. A ‗Sanctuary for Tiger‘ in Mundanthurai was declared in 1962 almost eleven years earlier than the launch of ‗Project Tiger‘ in the country in 1973. The Nilgiri Biosphere Reserve (NBR), a representative area of the Western Ghats (India) covers 5670 sq. km. It was set up in 1986 as the first Biosphere Reserve in India. Although the biosphere reserve programme was launched by the UNESCO in 1971, in India, the first biosphere reserve—Nilgiri biosphere reserve— came into being in 1986. Project Elephant was launched in February 1992 to assist states having free-ranging populations of wild elephants to ensure long term survival of identified viable populations of elephants in their natural habitats.

Q 90.B 



Species diversity is a term used to describe the measurement of biological diversity to be found in a specific ecological community. It includes the species richness or number of species to be found in an ecological community, the abundance (or number of individuals per species) and distribution or evenness of species. The tropics have greater biological diversity than temperate regions because o Speciation is generally a function of time. Unlike temperate regions subjected to frequent glaciations in the past, tropical latitudes have remained relatively undisturbed for millions of years and thus, had a long evolutionary time for species diversification. Hence option 3 is not correct. o Tropical environments, unlike temperate ones, are less seasonal, relatively more constant and predictable. Such constant environments promote niche specialisation and lead to a greater species diversity. Hence option 1 is not correct. o There is more solar energy available in the tropics, which contributes to higher productivity; this in turn might contribute indirectly to greater diversity. Hence option 2 is correct.

Q 91.C 



35

The first multi-modal terminals being constructed on the National Waterway-1 (River Ganga) as part of the World Bank-aided ‗Jal Marg Vikas Project‘ of the Inland Waterways Authority of India was inaugurated at Varanasi in November 2018. The Jal Marg Vikas project entails construction of three multi-modal terminals (Varanasi, Sahibganj and Haldia), two inter-modal terminals, five roll-on-roll-off (Ro-Ro) terminal pairs, new navigation lock at Farakka, West Bengal, assured depth dredging, integrated vessel repair and maintenance facility, differential global positioning system (DGPS), river information system (RIS) and river training. www.visionias.in

©Vision IAS

Q 92.D  

 





Peninsular India changed due to tectonic activity without affecting the original basement. As a part of Indo-Australian plate, it has been subjected to various vertical movements and block faulting. A rift valley is a linear shaped lowland between several highlands or mountain ranges created by the action of a geologic rift or fault. A rift valley is formed along a divergent plate boundary, a crustal extension or spreading apart of the surface, which is subsequently further deepened by the forces of erosion. Rivers Narmada, Tapti and Damodar flows through a rift valley. Hence option (d) is correct. The Narmada originates on the western flank of the Amarkantak plateau at a height of about 1,057 m. Flowing in a rift valley between the Satpura in the south and the Vindhyan range in the north, it forms a picturesque gorge in marble rocks and Dhuandhar waterfall near Jabalpur. After flowing a distance of about 1,312 km, it meets the Arabian sea south of Bharuch, forming a broad 27 km long estuary. The Tapi is the other important westward flowing river. It originates from Multai in the Betul district of Madhya Pradesh. It is 724 km long and drains an area of 65,145 sq. km. Nearly 79 per cent of its basin lies in Maharashtra, 15 per cent in Madhya Pradesh and the remaining 6 per cent in Gujarat. The Damodar occupies the eastern margins of the Chotanagpur Plateau where it flows through a rift valley and finally joins the Hugli. The Barakar is its main tributary. Once known as the ‗sorrow of Bengal‘, the Damodar has been now tamed by the Damodar Valley corporation, a multipurpose project.

Q 93.A 



The Zero Waste International Alliance has been established to promote positive alternatives to landfill and incineration and to raise community awareness of the social and economic benefits to be gained when waste is regarded as a resource base upon which can be built both employment and business opportunity. The Zero Waste International Alliance: o initiates and facilitates research and information sharing for the promotion of Zero Waste o builds capacity to effectively implement Zero Waste o sets standards for evaluating the achievement of Zero Waste One of the primary goals of organizing the Zero Waste International Alliance in 2002 was to establish standards to guide the development of Zero Waste in the world.

Q 94.C Isotherms  The lines drawn on maps joining the places of equal temperature reduced to sea level are called isotherms. It is necessary to reduce the actual temperatures of all places at sea level before drawing isotherms. It is thus, obvious that isotherms do not represent the real temperature of the places through which they pass rather they show the temperature of the places at sea level. This is why the isotherms maps are not useful for farmers because they need real temperature of particular places for growing crops.  Normally isotherms run east-west and are generally parallel to latitudes. This trend shows strong control of latitudes on the horizontal distribution of temperature.  Generally, isotherms are straight but they bend at the junction of the continents and oceans due to differential heating and cooling of land and water.  Isothermal lines are more irregular in the northern hemisphere because of the large extent of continents but they are more regular in the southern hemisphere due to over of oceans.  Isotherms are generally closely spaced in the northern hemisphere but they are more regular and widely spaced in the southern hemisphere. Hence statement 1 is correct. The closely spaced isotherms denote rapid rate of change of temperature and steep temperature gradient. On the other hand, widely spaced isotherms indicate a slow rate of change of temperature change and low-temperature gradient.  Two isotherms can never meet (intersect) despite the closeness between them. As the intersection of two isotherms would mean that a particular region has two different temperatures at one point in time, which is impossible. Hence statement 2 is correct.  On an average , isotherms trending from land towards the ocean bends equatorward during summer ad poleward during winter. On the other hand isotherms trending from the oceans to continents bend poleward during summer and equatorward during winter.

36

www.visionias.in

©Vision IAS

Q 95.C 





The project, Securing Livelihoods, Conservation, Sustainable Use and Restoration of High Range Himalayan Ecosystems project (SECURE Himalaya), was launched by Ministry of Environment, Forest, and Climate Change of the Government of India and the United Nations Development Programme (UNDP). It is part of the Global Wildlife Program, a seven-year program funded by the GEF and led by the World Bank, that was developed as a response to the growing crisis of illegal trafficking in wildlife. Hence, statement 2 is correct. The "Secure Himalaya" plan intended to conserve the snow leopards by protecting their habitats and improve the ecology of Himalayan ranges and lives of the mountain communites, all among the worst victims of climate change. Hence, statement 1 is correct. It is being implemented by the Government of India in partnership with UNDP to sustain critical ecosystem services and to conserve vulnerable snow leopards by securing community livelihoods, enhancing enforcement, strengthening community institutions, and improving knowledge, advocacy and information systems for promoting landscape-based conservation approaches. This project will contribute to the Global Snow Leopard and Ecosystem Protection Program (GLSEP), an effort to conserve the species in the 12 range countries, including India.

Q 96.C 



Statement 1 is correct: Himachal and Uttarakhand Himalayas lies approximately between the Ravi in the west and the Kali (a tributary of Ghaghara) in the east. It is drained by two major river systems of India, i.e. the Indus and the Ganga. Tributaries of the Indus include the river Ravi, the Beas and the Satluj, and the tributaries of Ganga flowing through this region include the Yamuna and the Ghagara. The two distinguishing features of this region from the point of view of physiography are the ‗Shiwalik‘ and ‗Dun formations‘. Duns are the longitudinal valleys present in the Southernmost part of Western Himalayas and Himachal and Uttarakhand Himalayas. Some important duns located in this region are the Jammu Dun, Pathankot Dun, Chandigarh-Kalka dun, Nalagarh dun, Dehra Dun, Harike dun and the Kota dun, etc. Dehra Dun is the largest of all the duns with an approximate length of 35-45 km and a width of 22-25 km Statement 2 is correct: The Dooars or Duars are the alluvial floodplains in northeastern India that lie south of the outer foothills of the Himalayas and north of the Brahmaputra River basin. This region is about 30 km wide and stretches over about 350 km from the Teesta River in West Bengal to the Dhansiri River in Assam. Duars are formed in the Darjeeling and Sikkim Himalayas which are used for the development of tea gardens.

Q 97.B 



37

The Livestock Census has been conducted in the country periodically since 1919-20. The Livestock Census covers all domesticated animals and its headcounts. So far 19 such censuses have been conducted in participation with State Governments and UT Administrations. The 20th Livestock Census was conducted in participation with all States and Union Territories. It is an initiative of the Department of Animal Husbandry & Dairying under Ministry of Fisheries, Animal Husbandry & Dairying. Hence, statement 1 is not correct. As per the Census -: o The total Livestock population in India rises 4.6 per cent to nearly 536 million as compared to the previous Census. o The percentage of different domesticated animals in the total Livestock are as under -: Cattle (35.94%), Goat (27.80%), Buffaloes (20.45%), Sheep (13.87%) and Pigs (1.69%). Hence, statement 2 is correct. o The population of cows in the country has risen by 18 per cent in the last seven years, while that of oxen dipped by 30 per cent. o Besides, there was a spectacular 16.8 per cent increase in the poultry population in the country to 851.81 million, mainly on account of a 46 per cent rise in backyard poultry birds, whose numbers have gone up to 317 million o Among the States, Uttar Pradesh has the highest number of livestock of 67.8 million (68.7 million in 2012), followed by Rajasthan 56.8 million (57.7 million), Madhya Pradesh: 40.6 million (36.3 million) and West Bengal: 37.4 million (30.3 million). Hence statement 3 is not correct. www.visionias.in

©Vision IAS

FREE BOOKS, NOTES & VIDEOS FOR CIVILSERVICES

EBOOKS & MAGZINES

UPSC PRELIMS MATERIALS

USPC MAINS MATERIALS

VIDEO FOR CIVILSERVICES

DAILY NEWSAPERS

SECUREIAS

UPSC PRELIMS TESTSERIES

UPSC MAINS TESTSERIES

DELHI STUDENTS

CIVILSERVICES BOOKS

OPTIONAL SUBJECTS BOOKS, STATE PCS, SSC, BANKING TEST SERIES, VIDEOS & NOTES BOOKS, TESTS VIDEOS & NOTES 1.GEOGRAPHY 2.HISTORY

1.UPPSC 4.IBPS

3.MATHEMATICS 4. SOCIOLOGY

ENGINEERING BOOKS & MATERIAL 1. IES 2. GATE 3. IFoS

5.PUBLIC ADMINISTRATION 6. POLITICAL SCIENCE

4. COMPUTER SCIENCE 5. MECHINICAL ENGINEERING

7. ECONOMICS

OTHER TELEGRAM CHANNELS

8 PHYSICS 9 COMMERCE ACCOUNTANCY 10 ANTHROPOLOGY 11 LAW 12 PHILOSOPHY 13 CHARTERED ACCOUNTANTANCY 14 MEDICAL SCIENCE

1 GOVERNMENT JOBS 2 LEARN YOGA & MEDITATION 3 LEARN ENGLISH 4 BEST DELAS & OFFERS 5 IAS HINDI BOOKS 6 PDFs FOR ALL EXAMS 7. WORLD DIGITAL LIBIRARY

1.CHENNAI STUDENTS 2.BANGLORE STUDENTS

2.SSC 3.MPSC 5.RAS & RPSC

3. CURRENT AFFAIRS

CONTACT FOR ADVERTISEMENT IN ABOVE CHANNLES ADMIN1:

ADMIN2:

Q 98.B 







The ocean absorbs vast quantities of heat as a result of increased concentrations of greenhouse gases in the atmosphere, mainly from fossil fuel consumption. The Fifth Assessment Report published by the Intergovernmental Panel on Climate Change (IPCC) in 2013 revealed that the ocean had absorbed more than 93% of the excess heat from greenhouse gas emissions since the 1970s. This is causing ocean temperatures to rise. The average global sea surface temperature – the temperature of the upper few metres of the ocean – has increased by approximately 0.13°C per decade over the past 100 years. Ocean warming leads to deoxygenation – a reduction in the amount of oxygen dissolved in the ocean – and sea-level rise – resulting from the thermal expansion of sea water and continental ice melting. The rising temperatures, coupled with ocean acidification (the decrease in pH of the ocean due to its uptake of CO2), affect marine species and ecosystems and, consequently, the fundamental benefits humans derive from the ocean. Marine fishes, seabirds and marine mammals all face very high risks from increasing temperatures, including high levels of mortalities, loss of breeding grounds and mass movements as species search for favourable environmental conditions. Coral reefs are also affected by increasing temperatures which cause coral bleaching and increase their risk of mortality. Hence only statement 1 is not correct.

Q 99.C 





38

Mediterranean type of climate o Mediterranean climate, is characterized by hot, dry summers and cool, wet winters and located between about 30° and 45° latitude north and south of the Equator and on the western sides of the continents. o During summers, the belt of the influence of the Westerlies is shifted polewards, and the rain-bearing tradewinds are likely to be off-shore. In the winter months, the Westerlies belt shifts equatorward and the Mediterranean regions are under the influence of on-shore Westerlies. o The anticyclones cause the surrounding air to diverge and descend leading to a clear sky. o Climatic changes are profound in the Mediterranean climatic region with rains alternating with warm sunny days during the winter seasons. o The Mediterranean lands are also known as the world‘s orchard lands. A wide range of citrus fruits such as oranges, lemons, limes, citrons and grapefruit are grown. The fruit trees have long roots to draw water from considerable depths during the long summer drought. o Rainfall varies from year to year and does not fall evenly. The rains do not arrive yearly at the same time or within the same interval. o Temperatures also vary from year to year with winter temperatures falling to as low as zero and may rise to as high as over 50 degree C in arid areas. China type of climate o It is the most typical of the warm temperate eastern margin climate. o In summer due to intense heating of the continental interiors of the heart of Asia including Tibet, a low-pressure system is set in which attracts the tropical Pacific air stream o This is witnessed as the South-East monsoon in the region. o In winter, there is an intense high pressure over Siberia and the continental polar air stream flows outwards as the North-West Monsoon, bitterly cold and very dry. o There is little rain but there is considerable snow. o The region also experiences intense tropical cyclones called typhoons that originate in the Pacific Ocean and are most frequent in the late summer. The Temperate Continental (Steppe) Climate o Their climate is continental with extremes of temperature. o Summers are very warm. o Winters are very cold in the continental steppes of Eurasia o In contrast, the steppe type of climate in the southern hemisphere is never severe o The annual range of temperature is great, a direct result of continentality. o In its continental position, the annual precipitation of the Steppe Climate can be expected to be light. o The average rainfall may be taken as about 20 inches, but this again varies according to location from 10 inches to 30 inches. www.visionias.in ©Vision IAS



The Hot Desert and Mid-latitude Desert Climates o The hot deserts lie astride the Horse Latitudes or the Sub Tropical High Pressure Belts where the air is descending, a condition least favourable for precipitation of any kind to take place. o The relative humidity is extremely low, decreasing from 60 per cent in coastal districts to less than 30 per cent in the desert interiors. o Rain normally occurs as violent thunderstorms of the convectional type. o The deserts are some of the hottest spots on earth and have high temperatures throughout the year. o There is no cold season in the hot deserts and the average summer temperature is around 86ºF.. o The annual range is 44ºF. o The diurnal range of temperature in the deserts is high.

Q 100.C  The Good Governance Index (GGI) is a uniform tool across states to assess the status of governance and the impact of various interventions taken up by the State Government and UTs. The objectives of GGI are to provide quantifiable data to compare the state of governance in all states and UTs, enable states and UTs to formulate and implement suitable strategies for improving governance and shift to result-oriented approaches and administration. The GGI has been designed & developed by Centre for Good Governance. Hence statement 1 is correct.  The GGI takes into consideration ten sectors: o Agriculture and Allied Sectors, o Commerce & Industries, o Human Resource Development, o Public Health, o Public Infrastructure & Utilities, o Economic Governance, o Social Welfare & Development, o Judicial & Public Security, o Environment and o Citizen-Centric Governance.  These ten Governance Sectors are measured on total 50 indicators.  Tamil Nadu has bagged the top position in the composite ranking for good governance index (GGI), followed by Maharashtra and Karnataka, according to data provided by the Personnel Ministry. Hence statement 2 is correct.  Centre for Good Governance o It was established in October 2001 by the then Government of Andhra Pradesh (GoAP) in collaboration with the Department for International Development (DFID) and the World Bank to help it achieve the State‘s goal of Transforming Governance. o It undertakes action research, provides professional advice, and conducts change management programmes for government departments and agencies to enable a successful implementation of their reform agenda. CGG works closely with policy-makers like Ministers, senior officials, management experts, institutions and other stakeholders, especially citizens towards building people centric governance practices.

Copyright © by Vision IAS All rights are reserved. No part of this document may be reproduced, stored in a retrieval system or transmitted in any form or by any means, electronic, mechanical, photocopying, recording or otherwise, without prior permission of Vision IAS. 39

www.visionias.in

©Vision IAS

Related Documents

Vision: Www.visionias.in
January 2021 1
New Vision
January 2021 1
Vision: Www.visionias.in
January 2021 1
Vision: Www.visionias.in
January 2021 2
Vision: Governance
January 2021 2

More Documents from "dndamalia"

Vision: Www.visionias.in
January 2021 1